MCAT Psychology 12/30/15 (From test 3)

Pataasin ang iyong marka sa homework at exams ngayon gamit ang Quizwiz!

Integrative reminiscence

refers to the process by which older people may take stock of their lives and come to terms with previously unresolved conflicts

In statistics, ________ is the extent to which the results are accurate and consistent over time (repeated measurements produce the same results) reliability or precision?

reliability

______ is the process which people learn to be proficient members of society; a lifelong process where people learn the attitudes, values, and beliefs that are reinforced by a particular culture

socialization

________ learning involves an individual learning by observing another individual.

vicarious

What are the parts of the basal ganglia?

*situated at the base of the forebrain 1. striatum 2. globus pallidus 3. substantia nigra 4. nucleus accumbens 5. subthalamic nucleus **It is associated with voluntary movement, procedural learning, and emotions

Huntington's Disease

- autosomal dominant mutation - is a fatal genetic disorder that causes the progressive breakdown of nerve cells in the brain - It deteriorates a person's physical and mental abilities during their prime working years and has no cure - is known as the quintessential family disease because every child of a parent with this disease has a 50/50 chance of carrying the faulty gene

Depth-of-Processing Model vs the Dual-Store Model (i.e. Dual Coding Hypothesis)

1) Dual-of-Processing Model - holds that information is transferred from short-term memory (STM) to long-term memory (LTM) when it is processed at deeper levels of analysis 2) Dual Coding Hypothesis - holds that information is moved from STM to LTM when it is maintained in STM for a sufficiently long period of time via rehearsal - involves associated images

Jean Piaget's stages:

1. Object permanence develops during Piaget's first stage, the sensorimotor stage During all of the following stages, according to Piaget's theory of cognitive development, children are using the object permanence skill they previously developed. 2. During pre-operations, children think more symbolically as they develop memory and imagination 3. The next stage is concrete operations, when children begin to exhibit logical reasoning 4. followed by Piaget's final stage, formal operations, when children are able to consider abstract relationships and concepts

Freudian Defense Mechanisms:

1. The Freudian defense mechanism of projection involves disowning unwanted characteristics of the self, and instead attributing them to others. At its most pathological, it can involve delusional aspects, but it can also function in a more reality-based manner. This would generally be the basis for the explanation of prejudice and/or abusive behavior exhibited by males with an insecure gender identity towards male gender-nonconformers 2. Sublimation is a mature defense in which a person transforms negative thoughts or feelings into positive actions or outlets; for example, sublimating the desire to harm others into a pro-social desire to defeat business rivals. 3. Denial is the refusal to accept reality, such as when a person responds to very bad news by refusing to believe the information is true. It is a very primitive defense, and would not involve reacting to characteristics of one's self by seeing them in others 4. Splitting is also a primitive defense, which is described as separating positive and negative characteristics and impulses from each other by segregating them into all good and all bad categories. While splitting can contain projection, the segregation of impulses into all good is not described in the passage (choice D is wrong).

When a person is participating in cognitive dissonance, what are three ways they can cope with it:

1. by lowering the importance of one of the discordant factors (i.e. downplaying something) 2. changing one of the discordant factors (i.e. justify breaking off the relationship with the meat eater) 3. adding consonant elements (i.e. attempting to convince partner to eat less meat)

Different types of attachment styles: 1. Secure attachment 2. Anxious-resistant insecure attachment style 3. Disorganized attachment style 4. Anxious-avoidant attachment style

1. is promoted by a responsive and available parent and is characterized by a child who is able to explore their environment with confidence because they know the parent will be there should they need them for comfort. A child confidently exploring and joining in different activities at a birthday party demonstrates a secure attachment style. The child seeking his mother's aid during a stressful time, such as being injured, and the parent being available to comfort and soothe him, further displays secure attachment 2. Reluctance to explore and instead clinging and remaining by the mother's side; the child is uncertain about the parent's availability because of previous separations. Because of this fear of being abandoned, he or she stays close and clings to the mother 3. Odd or awkward behaviors such as extreme shifts between proximity seeking and avoidance when separating or reuniting with the parent; A child who, upon being reunited with the parent, initially runs toward him/her with open arms and quickly shifts and runs away instead, illustrates an extreme shift between proximity seeking and avoidance. Disorganized attachment is usually the result of the child experiencing trauma, such as being physically, emotionally, or sexually abused by the parent. In this instance, the child sees the parent as both frightening and their only source of comfort, which causes them great confusion, thus, the confused, proximity seeking and avoidance behavior 4. results from the child being constantly rejected and rebuffed by the parent. These children learn to be independent early on, only depending on themselves because they learn that they can't rely on the parent because they are never available. A child exploring and climbing the jungle gym to dangerous heights displays this early independence. The child's knowledge about the unavailability or responsiveness from the parent further provides evidence of an anxious-avoidant attachment style (choice D is wrong).

Types of theories/ perspectives/ other: 1. Canonic perspective 2. Illusory contours 3. Echoic memory 4. Object relations theory

1. refer to prototype images that represent particular concepts or objects for an individual 2. refer to a related but distinct aspect of visual perception, namely detection of borders that demarcate (separate) a familiar shape or pattern 3. facet of perception, but refers specifically to short-term auditory memory capacity 4. does not pertain to visual perception; is an outgrowth of psychodynamic theory that focuses on early child-parent interaction and an individual's internal representation of relationship

1. difference threshold 2. absolute threshold 3. just noticeable difference (JND)

1. the amount of energy that needs to be added to or subtracted from the standard stimulus before there appears to be a difference 2. the amount of stimulus energy needed for a person to perceive it at all 3. is the same value as the difference threshold

In addition to regulating executive function, the frontal lobe is also responsible for: I. initiating voluntary movement. II. regulating hunger, thirst, and sex drive. III. regulating breathing, heart rate, and blood pressure. A) I only B) II only C) I and II only D) I and III only

A Item I is true: the primary motor cortex, which is responsible for initiating voluntary movement, is located in the frontal lobe (choice B can be eliminated). Item II is false: hunger, thirst, and sex drive are regulated by the hypothalamus, located in the diencephalon, just above the brain stem (choice C can be eliminated). Item III is false: breathing, heart rate, and blood pressure are regulated by the medulla oblongata, located in the hindbrain (choice D can be eliminated; choice A is correct).

Alcohol withdrawal syndrome occurs when an individual with a dependence on alcohol suddenly limits or stops alcohol consumption. Symptoms of withdrawal can be very dangerous, including seizures, uncontrollable shaking of the extremities, and other nervous system issues. What is the most plausible mechanism of action for these physical symptoms? A) Chronic alcohol consumption causes down regulation of GABA receptors, leading to a reduction in CNS inhibition, and excito-neurotoxicity B) Long-term alcohol abuse stimulates the autonomic nervous system, causing tremors C) cessation of alcohol consumption leads to a reduction in dopamine production in nucleus accumbens D) alcohol is a hallucinogenic, and withdrawal acts by relaxing, disinhibiting, and amplifying sensory information

A alcohol is a depressant and inhibits neural activity. GABA receptors in CNS respond to GABA, an inhibitory NT. Alcohol acts on GABA receptors, inhibiting neuronal signaling Chronic alcohol consumption causes a down-regulation of GABA receptors; therefore once the artificial depressant (alcohol) is removed from the system, the CNS no longer has an inhibitory influence, and excess-neurotoxicity occurs, which can result in seizures and tremors While alcohol consumption does promote dopamine release in the nucleus accumbens (which stimulates the reward pathway in the brain and helps to explain why alcohol is addictive), and cessation of alcohol consumption would surely lead to a decrease in dopamine, this does not explain the physical symptoms of withdrawal described in the question stem **GABA receptors are channel receptors. This means that when GABA binds to them, they change shape slightly to allow ions to pass through their central channel. This channel mainly allows negatively charged chloride ions to enter the neuron, thus reducing its excitability. Because of this property of the GABA channel receptor, GABA is classified as an inhibitory neurotransmitter, as opposed to excitatory neurotransmitters, such as glutamate, which augment the nerve impulses in the neuron.

Regression coefficient

A regression coefficient is a way to characterize the relationship between the independent and the dependent variable, and graphically is represented by the slope of the line that represents the rate of change of one variable as a function of changes in the other

In the semantic network model, what determines the strength of a connection between a node and an association? A) How frequently and deeply connections are made B) How closely the node and association are explicitly related C) How many links there are between the node and the association D) How many state-dependent cues have been created between the node and the association

A. According to the semantic network model, the strength of a connection between a node and an association is related to how frequently and how deeply the connections are made (choice A is correct). The explicit relationship between the node and the association is meaningless unless that connection is constantly reinforced (choice B is wrong). Separate links between the node and the association—suggesting a more circuitous route—is indicative of the spreading activation pattern (choice C is wrong). State-dependent cues, in which familiar locales are used to trigger memories, do not determine the strength of the connection between a node and an association unless again, they are used with great frequency (choice D is wrong). *Semantic memory = memory for factual information

Chemical messengers utilized for communication among members of the same species are called: A) exogenous hormones. B) endogenous hormones. C) pheromones. D) mating rituals

C Pheromones, while poorly understood, are frequently employed by various social species to communicate with each other (choice C is correct). Hormones, whether exogenous (created outside the body) or endogenous (created inside the body), are employed for communication and regulation within a given organism, not between organisms (choice A and choice B are wrong). Mating rituals, while utilized for communication among members of the same species, are not chemical messengers (choice D is wrong).

There are numerous broad categories of mental disorders. Personality disorders involve enduring maladaptive behaviors and traits that deviate from the social norms. Pathological, or compulsive, liars best fit the description of which cluster B disorder? A) Antisocial personality disorder B) Borderline personality disorder C) Histrionic personality disorder D) Schizotypal personality disorder

A. Despite the perspective described in the passage, there are also instances where failure to tell the truth is not purposeful, and thus does not contribute to social function. In these cases, lying can be associated with certain personality disorders. The personality disorders (PDs) are separated into three categories, with cluster B disorders being those associated with emotional, dramatic, and attention-seeking behaviors. Cluster B includes antisocial, borderline, histrionic, and narcissistic PDs (choice D can be eliminated). Compulsive lying is a problematic behavior associated with antisocial PD (choice A is correct). Overall, antisocial PD is characterized by a history of serious behavior problems beginning in adolescence. Borderline PD is characterized by enduring or recurrent instability in impulse control, mood, and image of self and others (choice B is wrong). Histrionic PD is characterized by a strong desire to be the center of attention (choice C is wrong). Finally, although not a cluster B disorder, schizotypal PD is characterized by several traits that cause problems interpersonally. Lying might be classified as a problematic behavior in interpersonal relationships, but schizotypal PD is categorized as a cluster A disorder.

A pharmaceutical company conducting clinical trials on a new medication for depression reports that the medication should be brought to market based on survey results. Which of the following, if true, would most compromise the company's conclusion? A) A researcher responsible for obtaining consent knew which participants were in the control and experimental groups. B) The research team did not conduct a regression analysis on the surveys to check for the effect of income on depression. C) Participants were offered fixed compensation for participating in the study. D) The drug did not outperform a generic alternative already on the market.

A. If a researcher knew which participants were receiving treatment and which were in a control group, then the study was not conducted in a double-blind manner. A double blind study requires researchers performing the experiment or working with participants to be unaware of which participants are receiving which treatment. Among other problems, such a design would risk introducing a placebo effect, a measurable improvement in health not due to the medication, but rather due to self-fulfilling prophecy, the phenomenon by which an expectation can influence a person's behavior such that the expectation is more likely to happen (choice A is correct). The fact that a drug did not outperform a generic alternative might be problematic, but would not compromise the conclusion that the drug should be brought to market since its efficacy would not be called into question by poor research design (choice D is wrong). If compensation were fixed, thus not dependent on responses to the survey, this would not impact conclusions about the efficacy of the drug (choice C is wrong). The company did not purport to test the impact of income on depression; thus, this would not impact their conclusion (choice B is wrong).

In the classic Little Albert experiment, Albert was conditioned to fear white, fluffy animals after they were paired with a sudden loud noise. Which of the following correctly labels the responses and stimuli? A) Animals (CS), loud noise (UCS), fear of loud noises (UCR), fear of animals (CR) B) Animals (UCS), loud noise (CS), fear of loud noises (UCR), fear of animals (CR) C) Animals (CR), loud noise (UCR), fear of loud noises (CS), fear of animals (UCS) D) Animals (UCR), loud noise (CR), fear of loud noises (UCS), fear of animals (CS)

A. In general, most humans (especially young children) and animals would fear a loud, unexpected noise. Consequently, the noise is an unconditioned stimulus (UCS), and the fear response an unconditioned response (UCR). When the noise is paired with a seemingly neutral stimulus (a white, fluffy animal), the brain ties the fear of the noise to the animal. Consequently, the animal becomes a conditioned stimulus (CS), and the fear in response to the white, fluffy animal becomes a conditioned response (CR; choice A is correct). Any other combination of labels and stimuli/responses is incorrect (choices, B, C, and D are wrong).

Which of the following would provide the strongest support for the heritability of schizophrenia? A) A pair of monozygotic twins adopted into different non-schizophrenic families, both presenting schizophrenic symptoms. B) A pair of dizygotic twins reared together, both presenting schizophrenic symptoms. C) A pair of monozygotic twins reared together, both presenting schizophrenic symptoms. D) A child that was born to and reared by schizophrenic parents developing schizophrenic symptoms.

A. Monozygotic twins share 100% of their genes. For twins adopted into different non-schizophrenic families and reared apart, one can assume that the onset of schizophrenic symptoms was caused by their shared genetic makeup, rather than shared environmental influences (choice A is correct). Monozygotic and dizygotic twins reared together would grow up in the same environments. This might make it difficult to determine whether the onset of schizophrenia was caused by genetic or environmental factors (choices B and C are wrong). A child born to and reared by schizophrenic parents could develop schizophrenia as a result of either genetic factors or maladaptive environmental influences (i.e., being raised by schizophrenic parents). Therefore, it would be difficult to distinguish the effects of the child's genotype form those of the environment (choice D is wrong).

According to Skinner, in terms of behavior modification, punishment: A) is typically only effective as long as the punishment is present. B) is the single most effective form of behavior modification. C) is more effective when applied in an inconsistent manner. D) causes misbehavior.

A. Skinner asserted that punishment is not an ideal form of behavior modification because it is only effective so long as the punishment, or threat of punishment, is present. For example, every time your cat jumps onto the counter, you squirt it with water and it jumps down. But when the threat of being sprayed is not present (e.g. you are in another room or not home), your cat is free to jump on the counters, and thus is learning to manipulate the misbehavior in order to avoid the punishment (choice A is correct). For those very reasons, punishment is not the single most effective form of behavior modification (choice B is wrong). Punishment is most effective when applied consistently, not inconsistently (choice C is wrong). Finally, there has been no definitive evidence that the use of punishment causes misbehavior, though it can lead to sneakier forms of the misbehaviors it is being used to extinguish (choice D is wrong).

Social identity theorists think that the greater the identification with the in-group: A) the more likely in-group bias is to occur. B) the less likely in-group bias is to occur. C) the less likely one is to seek out membership in other in-groups. D) the more likely one is to seek out membership in other in-groups.

A. Social Identity research indicates that strong in-group identification often leads to stronger in-group bias (the preference for other in-group members) and consequent out-group derogation, defined as discrimination against those not in the in-group due to perceived threat (choice A is correct and choice B is wrong). Social identity theorists hold that people's strength of identity with their in-groups significantly impacts their attitudes toward incompatible out-groups. However, individuals could potentially be members of multiple, non-contradictory in-groups (alumni groups, sports fandoms, etc.), and their decisions to join such groups are not necessarily influenced by the strength of identification with groups to which they already belong (choices C and D are wrong).

Which subcortical structure is the most likely to be involved in object permanence? A) Hippocampus B) Thalamus C) Medulla oblongata D) Hypothalamus

A. The object permanence task, according to the passage, is a measure of working memory. The subcortical structure most involved in working memory is the hippocampus (choice A is correct). The thalamus is involved in relaying sensory and motor signals to and from the cortex and regulating sleep and consciousness (choice B is wrong). The medulla oblongata controls autonomic processes like respiration and heart rate (choice C is wrong). The hypothalamus is involved in metabolic processes and hormone release (choice D is wrong).

Surgical severing of the corpus callosum is a potential treatment for severe epilepsy. A post-surgical epileptic is in the car, on the way home from the hospital. Which of the following would be expected when the person is looking straight ahead? A) The person will be able to verbalize the names of streets he sees on the right side of the road but not the names of streets on the left side of the road. B) The person will be able to verbalize the names of streets he sees on the left side of the road but not the names of streets on the right side of the road. C) The person will not be able to verbalize the names of streets on either side of the road. D) The person will be able to verbalize the names of streets on both sides of the road, but they will appear upside down.

A. The right side of the visual field of both eyes is processed on the left side of each retina. These images are then routed by axons to the left cerebral hemisphere. Similarly, the left side of the visual field is reflected onto the right side of each retina and routed to the right cerebral hemisphere. Assuming the corpus callosum (CC) is needed for the two hemispheres to share information, the absence of a CC would mean that images delivered to the left hemisphere remain in the left hemisphere, and images delivered to the right hemisphere remain in the right hemisphere. Since language is produced in the left hemisphere, the person would thus be able to verbalize images captured in the left hemisphere, which are those from the right side of the visual field (choice A is correct; choice C is wrong). They would not be able to verbalize those images captured in the right hemisphere, which are those from the left side of the visual field (choice B is wrong). Additionally, the CC is not responsible for correcting the inverted image as it is apprehended by the retina. Rather, that is the function of the visual cortex (choice D is wrong).

When the unconditioned stimulus is no longer delivered with the conditioned stimulus, the subject's response to the conditioned stimulus diminishes. What is the term used to describe this phenomenon? A) Execution B) Habituation C) Elimination D) Extinction

D. The term extinction is used to describe the phenomenon in which response to the conditioned stimulus diminishes once it fails to be associated with the unconditioned stimulus (choice D is correct). Execution and elimination are not terms associated with classical or operant conditioning (choices A and C are wrong). Habituation refers to the phenomenon in which an individual becomes so used to a particular stimulus that he or she stops having a response altogether (for example, a florescent light bulb buzzes so constantly that people stop hearing it; choice B is wrong).

A individual's description of her behavior during her job earlier that day is an example of retrieval from: A) episodic memory. B) semantic memory. C) procedural memory. D) olfactory memory.

A. Episodic memory refers to memory of one's personal experiences (choice A is correct). Semantic memory is memory of concepts and information (choice B is wrong), while procedural memory is memory of how to execute motor tasks (choice C is wrong). Olfactory memory is not a term associated with models of long term memory. In the context of short term memory, it would refer to a sensory store from which one could recall what a given stimulus smelled like. It would not have relevance to recall of one's behavior (choice D is wrong).

H. M. was an American medical patient who suffered from severe seizures following a bicycle accident in his youth. In 1953, he was treated by the removal of approximately 2/3 of his left and right hippocampi, as well as parts of other brain structures. He subsequently suffered from what disorder? A) Anterograde amnesia B) Epilepsy C) Schizophrenia D) Procedural amnesia

A. H. M. suffered from complete anterograde amnesia following the removal of most of his hippocampi, meaning he could no longer form new episodic memories (choice A is correct). While H. M. did suffer from epilepsy, that existed prior to his surgery, and the seizures were largely eradicated by the treatment (choice B is wrong). Schizophrenia is believed to be an inherited disorder - although its expression is still impacted by the environment - that is not acquired later in life, though it manifests between late adolescence and early adulthood (choice C is wrong). H. M. was able to acquire new skills, though he was unable to remember learning them (choice D is wrong).

The following brain region is most likely to be implicated in the onset and degenerative progression of Alzheimer's disease: A) Hippocampus B) Thalamus C) Posterior occipital lobe D) Superior parietal lobe

A. The hippocampus is associated with the formation, storage, and retrieval of memories. Since Alzheimer's often profoundly affects a sufferer's ability to remember, this region is most likely to be implicated (choice A is correct). The thalamus is a relay center for sensory information that is important in sensation and perception of external stimuli. Since the primary symptom of Alzheimer's is memory loss, this is not the brain region most likely to be implicated in the onset and degenerative progression of AD (choice B is wrong). The posterior occipital lobe is critical in the perception of vision and organizing visual information (choice C is wrong). The superior parietal lobe has many functions, including integration of spatial information and maintenance of internal representations; however, none of these functions involves the primary symptoms of AD (choice D is wrong).

How might a theorist in the symbolic interactionist tradition explain deviance? A) Deviant behavior is a response to being labeled a deviant by others. B) Deviant behavior results from performing on the back stage when one should be performing on the front stage. C) Deviance is a failure to develop a looking-glass self. D) Deviance is a failure to resolve the identity versus role confusion dilemma in adolescence.

A. Labeling theory is a classic symbolic interactionist approach. It states that deviance is not inherent in the act itself, but is rather an attribution from others (such as police, family, or clergy; choice A is correct). Back stage and front stage dynamics are an aspect of the dramaturgical approach. If one acts on the back stage when she/he should be performing on the front stage, the audience might experience more than it otherwise would (more details about a person's motivation or inner thoughts, for instance), but this would not necessarily be a deviant act (choice B is wrong). A looking-glass self describes how people shape their self-concepts based on how they think others perceive them (choice C is wrong). The identity versus role confusion dilemma is one stage in Erikson's eight developmental stages. This is not part of the symbolic interactionist tradition (choice D is wrong).

How are cortisol levels and depression linked?

Elevated cortisol levels are directly correlated to decreased brain serotonin and an increase in depression

According to Lawrence Kohlberg, what is the highest level of morality that the typical average adult tends to achieve? A) She or he will acknowledge a sense of duty to uphold the laws of the land and maintain basic social conventions. B) He or she will strive for social justice—the greatest good for the greatest number. C) He or she will focus on the approval (and/or disapproval) of others, acting in accordance with how he or she will be viewed by his or her community or peer group. D) She or he will act in such a way as to avoid punishment at all costs.

A. Lawrence Kohlberg postulated six stages of moral development, but said that the average person generally did not pass beyond the fourth stage, in which he or she acknowledged a sense of duty to uphold the law and maintain social conventions (choice A is correct). Striving for social justice, where one acts for the greatest good for the greatest number, was the fifth stage in Kohlberg's schema and one that he proposed that few people reach (choices B is wrong). A focus on receiving approval from one's peers along with a need to conform is the third stage of development, usually seen in older children and adolescents (choice C is wrong). Acting to avoid punishment is the earliest stage of moral development, typical of very young children (choice D is wrong).

Which of the following is a negative symptom of schizophrenia? A) Avolition B) Disorganized speech C) Hallucinations D) Delusions

A. Negative symptoms of schizophrenia are defined as deficits in normal behavior, such as avolition, or lacking any motivation to initiate behavior. Someone experiencing avolition will remain in the same position for long periods of time unless moved by someone else (choice A is correct). Positive symptoms of schizophrenia are those that most individuals do not normally experience but are present in people with schizophrenia. Disorganized speech is speech which lacks a goal or coherence; this is a positive symptom (choice B is wrong). A hallucination is the perception of sensation (feeling, hearing or seeing) in the absence of an external stimulus, such as hearing a voice when no one is speaking or seeing something that no one else sees; this is a positive symptom (choice C is wrong). Delusions are false beliefs held with conviction in spite of contradictory evidence; this is a positive symptom (choice D is wrong).

The difference between normative social influence and informational social influence is: A) normative social influence is about people conforming for social acceptance, and informational social influence is about people conforming for "the right reasons" and "others know more." B) normative social influence is about people conforming for "the right reasons" and "others know more," and informational social influence is about people conforming for social acceptance. C) normative social influence is about people conforming to help normalize society, and informational social influence is people conforming in the interest of educating society. D) normative social influence occurs when people decide for themselves to conform, and informational social influence occurs when an authority figure, such as a political or religious leader, motivates the people to conform.

A. Normative social influence occurs when people conform because they desire social acceptance. Informational social influence occurs when people conform because they want to make ethically correct choices and also because they believe that others know more than they do (choice A is correct). Normative social influence does not occur out of a desire to be morally correct or to follow those more knowledgeable, nor does informational social influence occur when people desire social acceptance. Rather, it is the reverse (choice B is wrong). Normative social influence has nothing to do with normalizing society or establishing social norms. Similarly, informational social influence has nothing to do with informing or educating society (choice C is wrong). Normative social influence is not about people deciding for themselves when to conform. However, informational social influence could occur when an external authority figure tells the people it is time to conform because the people could decide that that authority figure either is morally correct or knows more than they do, the definition of informational social influence (choice D is wrong).

What aspect of the dialogic reading method described most aligns with B. F. Skinner's model of language acquisition? A) Repeating children's correct answers and encouraging and praising them. B) Asking "what" and other open-ended questions. C) Modeling good answers for the children. D) Making it into a game so children have fun.

A. Repeating children's correct answers and encouraging and praising them is part of Skinner's model of language acquisition by operant conditioning. He believed that language use, like any other behavior, is subject to conditioning and requires repetition and positive reinforcement (choice A is correct). Asking open-ended questions, part of this dialogic method, does not most align with Skinner's behaviorist methodology (choice B is wrong). Modeling good answers for the children would be more in keeping with Bandura's theory of modeling than with Skinner's idea of conditioning (choice C is wrong). Making the activity fun is simply part of the dialogic method to keep children engaged and motivated. It is not most aligned with Skinner's theory (choice D is wrong).

Stereotyping vs Discrimination vs Prejudice

A. Stereotyping - involves oversimplified ideas about groups of people, based on their race B. Prejudice - refers to the thoughts, attitudes, and feelings someone holds about a racial group that are not based on actual experience C. Discrimination - involves action toward a racial group

If the results showed that the participants in the Full Attention group scored higher than the Divided Attention group, this could have been caused by all of the following, EXCEPT: A) the second task was too long. B) the second task was too difficult. C) the second task was too similar. D) the second task required too much practice.

A. The length of a task is not one of the three main factors that determine whether a task will drain a person of resources when trying to multitask, since the resource model states that humans have a limited amount of resources available when trying to attend to multiple tasks. If the task was too long, it would not have necessary led to the FA group scoring higher (choice A is correct). Task difficulty is one of three factors associated with performance on multitasking according to the resource model. If the task was too difficult, the FA group might have scored higher (choices B is wrong). Again, if the second task was too similar to the first, the FA group might also have scored higher than the DA group (choice C is wrong). Similarly, if the second task required way too much practice, the FA group might have scored higher than the DA group (choice D is wrong).

In Erikson's psychosocial stages of development, which stage occurs when it involves children learning self-control, as well as having the need to explore, test limits, and make mistakes?

Autonomy vs. Shame

Which of these statements does not describe accurate criteria for PTSD, as described by the DSM-5? A. Persistent avoidance of memories, thoughts, feelings, or external reminders associated with the traumatic event(s) lasting for more than one month. B. Exposure to serious injury, sexual violence, or threatened or actual death lasting more than one month. C. The presence of memories, dreams, or dissociated reactions that are involuntary, intrusive, distressing, and recurrent lasting for more than one month. D. Negative, distorted, or exaggerated, beliefs, cognitions, or moods associated with the traumatic event that lasts for more than one month.

B All of these describe criteria which are necessary for an individual to be diagnosed with PTSD; however, one criterion does not have a duration requirement. The DSM-5 does not specify a specific amount of time (duration) that a traumatic event must be experienced.

Cortisol has a direct inhibitory effect on: A) posterior pituitary B) hypothalamus C) adrenal cortex D) glycogen metabolism

B the hypothalamic-pituitary axis (HPA) regulates the release of cortisol via negative feedback hypothalamus releases corticotropin releasing hormone (CRH), which stimulates the anterior pituitary to release ACTH, which in turn stimulates the adrenal cortex to release cortisol cortisol then inhibits the hypothalamus from releasing further CRH and the anterior pituitary form releasing more ACTH, thus having an inhibitory effect on both *cortisol has an indirect inhibitory effect on adrenal cortex *cortisol stimulates glycogen mobilization

What structure in the middle ear generates vibrations that match the sound waves striking it? A) Basilar membrane B) Tympanic membrane C) Cochlea D) Malleus

B tympanic membrane = eardrum; located in middle ear; generates vibrations that match the sound waves striking it vibrations generated in the tympanic membrane pass through small bones (malleus or hammer, incus or anvil, and stapes or stirrup); bones magnify incoming vibrations by focusing them onto a structure known as the oval window they then enter the cochlea, a fluid-filled spiral structure in the inner ear base of cochlea is lined with a long, fluid-filled duct known as the basilar membrane; sound waves passing along the basilar membrane cause it to move up and down, stimulating hair cells in the organ of the Corti; these hairs connect with the acoustic or auditory nerve

All of the following describe the adaptive value of emotions, EXCEPT: A) allowing for quick decisions in possibly threating situations. B) hindering performance under conditions of extreme arousal. C) influencing behaviors in a social context. D) moderating behavior through consideration of possible emotional outcomes.

B While the Yerkes-Dodson Law does propose that extreme emotional arousal will hinder performance (as will under-arousal), this is not an adaptive value of emotion (choice B is correct). Emotions can enhance survival by serving as a useful guide for quick decisions. A feeling of fear can be a valuable tool to indicate that a situation may be dangerous, and will prompt us to react more quickly (choice A is an adaptive value, and is therefore wrong). Emotion also plays a role in influencing individual behaviors within a social context; for example, embarrassment encourages social conformity. Additionally, in social contexts, emotions provide a means for nonverbal communication and empathy, allowing for cooperative interactions (choice C is an adaptive value, and is therefore wrong). Emotions influence our everyday choices by allowing us to consider our emotional responses to possible outcomes (choice D is an adaptive value, and is therefore wrong). *adaptive: It represents usefulness of a trait that can help an organism to survive in its environment.

a male kindergarten teacher experiences societal expectations for being a man and being a kindergarten teacher at the same time. ______ is an example of: A. Role Strain B. Role Conflict C. Role Exit D. Master Status

B. Role conflict happens when there are conflicting societal expectations for multiple statuses held by the same person Here: a man and a teacher Role Strain: conflicting situations on one status EX: When a bisexual female reports feeling social pressure to be either lesbian or straight, which causes her significant stress Here: one woman, but has "strain" on what her sexuality should be

Nicotine belongs to which class of psychoactive drugs? A) Opiates B) Stimulants C) Depressants D) Hallucinogens

B. Nicotine is a stimulant, and its overall effect is to speed up bodily functions (choice B is correct). Opiates belong to the class of psychoactive drugs called depressants. These work by slowing down bodily activity (choice A is wrong, choice C is wrong). Hallucinogens cause perceptual distortions, such as seeing colors or feeling moods of euphoria or terror (choice D is wrong).

When given a new diagnosis of AD, studies suggest that people respond in many different ways, often employing various defense mechanisms. Which of the following responses would be considered intellectualization? A) Denying the diagnosis by questioning the credentials of the physician and seeking a second opinion. B) Focusing on the details of the diagnosis and seeking out as much information as possible about the disease. C) Physically lashing out at family and loved ones. D) Trying hard to forget the diagnosis ever occurred by focusing on other things.

B. According to psychoanalytic theory, intellectualization is a defense mechanism that involves an individual dealing with her or his emotions by focusing on the intellectual aspects of the problem or issue (choice B is correct). Denial is a defense mechanism that involves an individual behaving as though the problem or issue doesn't exist (choice A is wrong); displacement is a defense mechanism that involves an individual taking out their frustration on others (choice C is wrong); repression is a defense mechanism that involves an individual trying to push the problem or issue into her or his subconscious, pretending it never happened (choice D is wrong). **Psychoanalytic theory was developed by Sigmund Freud **theory states that your personality is shaped by your unconscious thoughts, feelings, and memories; existence of unconscious is inferred from behaviors like dreams, slips of the tongue, posthypnotic suggestions, and free associations

From Passage: The McGurk Effect is a perceptual phenomenon that demonstrates an interaction between hearing and vision in speech perception. When an individual receives auditory information that is incongruent with the visual component of another sound, the brain perceives a third, different sound. For example, if the auditory sound of "Ba" is paired with a video demonstrating the lip movements for the sound "Ga," most people will perceive the sound "Da." This is an example of an auditory illusion. This shows that the accurate perception of information can involve the participation of more than one sensory system, called multimodal perception. In this case, the perception of sound in the McGurk Effect involves information from both the auditory system and the visual system. The McGurk Effect is an example of what? A) Weber's Law B) Bottom up processing C) Working memory D) Visual perception

B. Bottom-Up processing is a type of information processing based on incoming data from the environment to form a perception. When individuals encounter novel stimuli they are forced to engage in bottom-up processing and construct their perception of the stimuli from bits of raw sensory information. When considering the McGurk Effect, an individual is creating a perception from the raw visual and auditory stimuli he or she encounters in the environment. This is an example of bottom-up processing (choice B is correct). Weber's law states that the change in a stimulus that will be perceived as just noticeable is a constant ratio of the original stimulus; this law helps explain when we will and will not perceive a difference between two of the same stimuli (for example a quiet sound and the same sound at a slightly higher volume), but does not have anything to do with the McGurk Effect (choice A is wrong). Working memory is part of the information processing theory, however it is a component of information processing whereby an individual holds and manipulates information in their mind before that information is encoded into long-term memory, which is not integral to the McGurk effect (choice C is wrong). Lastly, visual perception is one way in which humans receive information from their environment. Visual perception is one of many ways individuals gather data from their surroundings that interact to influence perceptions. Although visual perception is a type of sensory perception that is involved in the McGurk Effect, the McGurk Effect also relies upon auditory perception to create the illusion (choice D is wrong).

Which is the least effective method to combat the stereotype threat that affected students' GPAs? A) Encourage self-affirmation unrelated to students' black identity. B) Emphasize students' race and confirm they are capable of meeting high academic standards. C) Provide black role models to the students. D) Educate black students about stereotype threat so they are able to recognize it.

B. Emphasizing a person's marginalized identity typically causes one to experience more stereotype threat, so emphasizing students' race is least effective to combat stereotype threat, even if assurance is provided that high standards can be met (choice B is correct). Encouraging self-affirmation unrelated to the marginalized identity, providing role models of the same marginalized identity to students of a marginalized identity, and educating students about stereotype threat and how to recognize it have all been shown to reduce the negative effects of stereotype threat (choice A is wrong, choice C is wrong, and choice D is wrong, respectively).

Which of the following is least likely to lead to social loafing? A) Lack of individual evaluation B) Working in cohesive groups C) Incommensurate compensation D) Submaximal goal setting

B. If the group is a cohesive unit, individuals are more likely to be concerned about the outcomes. This is especially true of in-groups, whereby the members identify with one another and are more likely to make an effort in order to ensure they meet the expectations of their fellow members (choice B is correct). If group members are not being evaluated on an individual basis, they tend to feel less accountable for their work. For example, if a group of five medical students were to receive a single grade for a project, each would likely feel less compelled to contribute than he or she would if group members were receiving individual grades (choice A is wrong). If group members are not receiving the same levels of compensation, their efforts will likely reflect such disparity. For instance, if one team member is receiving $5 for her efforts, she will probably minimize her efforts if she knows other participants are receiving higher payouts (choice C is wrong). Submaximal goal setting occurs when group members decrease their efforts because they do not feel that have to work as hard when others are working with them toward the same goal (choice D is wrong). Incommensurate = not in proportion or equal

In Albert Bandura's infamous "Bobo Doll" experiment, children watched an adult either play nicely with a doll, or play very aggressively with it. The children were then likely to imitate whichever scenario they had seen when given a doll of their own. Which of the following describes this phenomenon? A) Latent learning B) Observational learning C) Associative learning D) Hands-on learning

B. In the "Bobo Doll" experiments, children modeled their behavior after the adults they viewed playing with the dolls, which is a component of observational learning (choice B is correct). Latent learning is defined as learning that is not immediately expressed in an overt response, and that occurs without any obvious reinforcement (e.g. a rat carried through a maze will eventually learn how to run through the maze on its own; choice A is wrong). Associative learning is the type of learning that occurs in both classical and operant conditioning, wherein the individual learns the association between two stimuli (choice C is wrong). While certain teaching theories espouse the use of concrete manipulatives for children of certain ages, the term "hands-on learning" is a colloquial one and is not associated with any learning theories, per se (choice D is wrong).

Which of the following weight-loss strategies would a neobehaviorist endorse? A) A combination of group therapy and prescription medication to help people lose as much weight as possible in a short timeframe. B) For every five pounds a person loses, she is given a non-food reward such as money or praise/attention from others. C) Whenever a person gains weight, he is publicly criticized by others. D) A group exercise class in which the exercises become increasingly more difficult over time.

B. Neobehaviorists believe that behavior can be modified by rewards or punishments (choice A is wrong because it does not describe any type of reward or punishment). Therefore, if someone exhibits a desired behavior, providing a reward after that behavior will encourage the behavior to happen again (positive reinforcement). When a person exhibits an undesirable behavior, the application of a punishment will discourage that behavior from happening again (positive punishment). While choice B describes positive reinforcement, and choice C describes positive punishment, neobehaviorists believe that the most effective way to modify behavior is with positive reinforcement (choice B is correct, choice C is wrong). The implementation of a punishment (exercises getting harder) in response to a desired behavior (exercise) does not fit within neobehaviorist principles (choice D is wrong).

Since the McGurk Effect relies upon visual and auditory stimuli, which of the following brain structures are NOT involved in the processing of the information that produces the McGurk Effect? A) Occipital lobe B) Postcentral gyrus C) Lateral geniculate nucleus in the thalamus D) Cochlear nucleus

B. The postcental gyrus is a prominent structure in the parietal lobe. The primary function of the postcental gyrus is in the processing of the sense of touch, not the senses of sound or sight, which are part of the McGurk Effect (choice B is correct). The occipital lobe is the visual processing center of the brain, and contains the visual cortex, which would be involved in the visual processing of the McGurk Effect (choice A is wrong). The lateral geniculate nucleus in the thalamus is involved in the visual processing pathway of the brain. It is the primary relay center for visual information received from the retina of the eye, and therefore would be involved in the visual processing of the McGurk Effect (choice C is wrong). The cochlear nuclei are collections of neurons that receive input from the sound received by the cochlear nerve, and then outputs that information to the auditory cortex of the brainstem. As the cochlear nuclei are involved in auditory processing, they would be involved in the processing of the McGurk Effect (choice D is wrong).

A person sees a small grassy area containing flowers outside of a residential building. This person's brain draws on various memories of gardens and the scents of flowers and grass to interpret that this grassy area is a garden. This is an example of: A) bottom-up processing. B) top-down processing. C) superimposition. D) contextual reception.

B. Top-down processing is a Gestalt psychology term for interpreting sensory stimuli based on past experience and larger contextual information in order to slowly work down and figure out the precise nature of each stimulus (choice B is correct). Bottom-up processing is the Gestalt term for the reverse process, by which one starts with a small sensory stimulus and integrates progressively more contextual information to determine its nature (choice A is wrong). Superimposition is a process whereby one stimulus is layered on top of another (choice C is wrong). Contextual reception is simply another way of saying that one is receiving contextual stimuli, not necessarily even processing it (choice D is wrong).

The primary function of working memory is to: A) focus mental resources on only one part of the stimulus field. B) hold and manipulate information stored in short-term memory. C) encode stimuli picked up by the sensory organs. D) retain information for long periods of time.

B. Working memory is a part of the memory system that allows us to process and manipulate information held in short-term memory (choice B is correct). Focusing mental resources on a select portion of the stimulus field is known as selective attention (choice A is wrong). The part of the memory system that receives input from the sensory organs is referred to as sensory memory (choice C is wrong). The part of the memory system that retains information for prolonged periods of time is known as long-term memory (choice D is wrong).

One aspect of socialization is learning the patterns of normal behavior of one's social group. Certain norms are formal and strongly enforced, while others are merely customary ways of doing things that don't carry harsh penalties when violated. These customary ways of doing things are known as: A) mores. B) folkways. C) values. D) sanctions.

B. Folkways are conventional ways of acting within a certain culture. Violating them would make one look unusual, but the violation would not carry an official penalty (choice B is correct). Mores, on the other hand, are formal norms that are very strictly enforced because they protect basic tenets of society (treason, for example, violates mores of loyalty and patriotism; choice A is wrong). Values are a culture's standard for evaluating what is good and bad (choice C is wrong). Sanctions are the punishments for violating a norm (choice D is wrong).

Which of the following is NOT associated with a gestalt approach to visual perception problems? A) Symmetry B) Illusory contours C) Continuity D) Common fate

B. Gestalt psychology emphasizes the mental processes that perceive the "whole" of objects or situations. This school of psychology has been particularly influential in describing tendencies of human visual perception. Among these tendencies are symmetry, continuity, and common fate. Symmetry refers to the tendency to view objects with subtle asymmetries as symmetrical (choice A is true and therefore wrong), while continuity is the tendency to "see" a pattern continue even after it has physically terminated (choice C is true and therefore wrong). Finally, common fate is the tendency to visually group objects of the same category together (choice D is true and therefore wrong). The notion of illusory contours is a type of illusion whereby boundaries and forms are perceived by the mind even though they do not exist in the actual image. Though this is a tendency of human visual perception, it is not associated with the Gestalt school, per se (choice B is false and therefore correct).

A child who learns to avoid approaching her parent for help and comfort because she is always met with aggression by the parent develops an anxious-avoidant insecure attachment style. The child thus associates the parent with fear and begins to actively avoid interactions with the parent in order to avoid this emotional experience. This type of learning can also be explained by which operant conditioning principle? A) Modeling B) Avoidance learning C) Vicarious learning D) Role playing

B. In avoidance learning, an organism's behavior is reinforced by the termination or prevention of an aversive stimulus. In other words, the person is motivated to escape fear producing environmental stimuli or conditioned stimuli. In this instance, because the child is always met with anger when he/she encounters the parent, the parent becomes a conditioned stimulus that produces fear. The child thus begins to actively avoid interactions with the parent in an effort to avoid experiencing fear (choice B is correct). Observational learning, also known as modeling or vicarious learning, is a part of Albert Bandura's Social Learning Theory. Bandura believed that learning could occur via watching others. However, not all observed behaviors are effectively learned. In order for the modeling process to work, according to Bandura, the following factors had to be present: attention, retention, reproduction, and motivation. The child in question is avoiding the parent and does not appear to be motivated to reproduce the aggressive behavior he is observing (choices A and C are wrong). Role-playing is not an operant conditioning principle. Rather, it is an instance or situation in which one deliberately acts out or assumes a particular character or role. In psychology, it is a therapeutic technique designed to reduce conflict in social situations, in which participants act out particular behavioral roles in order to expand their awareness of differing points of view. Role-playing has also been the focus of many social psychology studies (choice D is wrong).

A researcher wants to challenge the hypothesis that OXTR polymorphism genotype moderates the effect of social support on stress reactivity. The researcher proposes that individuals with GG genotypes were therefore raised by parents with either GG or AG genotypes and that the effect is actually environmental. What is one way that this researcher might test her hypothesis? A) Twin study B) Adoption study C) Transgenesis D) Developmental study

B. In order to test her hypothesis, the researcher will need to separate the influence of nature (i.e. genes) and nurture (i.e. environment). An adoption study is the best way to do this. In an adoption study, researchers compare individuals with their genetic and adopted relatives to determine if they are more similar to one group or the other. If the researcher found that the effect shown in Figure 1 only persisted if individuals with the GG/AG genotypes were also raised by adopted parents with GG/AG genotypes, then she would argue that the effect is due to environmental, rather than genetic, differences. However, if the researcher found the effect shown in Figure 1 for all adopted individuals, regardless of their adopted parents' genotypes, this would be evidence for an effect of genetics. Therefore, an adoption study would be the best way to test this hypothesis and choice B is correct. In a twin study, researchers assess whether monozygotic (i.e. identical) twins are more similar than dizygotic (i.e. fraternal) twins to determine to what extent a behavior can be attributed to shared genes and to what extent it can be attributed to shared environment (choice A is wrong). Transgenesis is when a researcher either introduces a new gene or removes a gene from an animal's pre-existing genome in order to assess the effect of that gene in a tightly controlled experiment. Due to ethical concerns, it is highly unlikely that a human would ever be a participant in a transgenesis study (choice C is wrong). A developmental study would allow the researcher to assess the effect of genotype on behavior at many different points throughout a person's lifespan, but it would not allow her to assess whether the effect was due to entirely genetic or partially environmental influence (choice D is wrong).

Which of the below statements demonstrates the idea that mass media are agents of socialization? I. Young people's body image is affected by the bodies depicted in magazines II. News media coverage of sexual assault focuses primarily on the victim III. Social media enable online bullying A) I only B) I and II only C) II and III only D) I, II and III

B. Item I is true: The mass media present and perpetuate a body image that is valorized by society. Exposure to these valorized images can socialize young people to take on these body standards as ideal. This is a form of socialization (choice C can be eliminated). Item II is true: When news media coverage focuses primarily on the victim of the assault, women learn that it is their job to protect themselves from sexual assault, but perpetrators do not learn to stop assaulting women. Emphasizing women's victimhood in mass media is a form of gender socialization that carries with it messages about who to associate with, how much to drink, and how to dress oneself (choice A can be eliminated). Item III is false: While social media can be considered to be mass media, the actual socializing force in this case is that of the peer group; peer groups are merely using social media to socialize each other (choice B is correct, choice D can be eliminated).

Research has also considered the salaries of medical professionals, and the effects of compensation on overall life satisfaction. Reports of average earnings ranged from $156,000 to $315,000 with pediatricians reporting the lowest salaries and orthopedic surgeons and radiologists reporting the highest salaries. Based on this information, models of social stratification in the United States would consider the average anesthesiologist to be a member of which social class? A) Upper Class B) Middle Class C) Working Class D) Lower Class

B. Most sociologists consider socioeconomic status (SES) when discussing social stratification. This measure is often defined in terms of power, prestige, and wealth due to the common interrelationship among these factors. The reported salaries of specialists range from $157,000 to $316,000 for the listed lowest and highest earners, respectively. The average anesthesiologist is then expected to earn an intermediate amount (the research reports an annual average of approximately $310,000). While estimates differ, this earning potential alone suggests that anesthesiologists are middle class, perhaps upper middle class (choice B is correct). Middle class: $46,000 to the minimum earnings representing the upper class (this definition varies); 40 percent of the population is considered middle class. Despite the high earnings of these medical specialists in comparison to the national average, it is a common misconception that these professionals are "upper class"; according to most models of social stratification, the income range for upper class is much higher. Upper class: range from millions to billions; a mere 3 percent of the population are considered upper class (choice A is wrong). There are medical professionals who earn comparable amounts when accounting for additional income and wealth; for example, Dr. Mehmet Oz, a famous cardiothoracic surgeon who hosts a television program focusing on medical issues and personal health (estimated annual earnings of $4 million). However, the information provided considers the average anesthesiologist. Working class: $19,000 to $45,000; 30 percent of the population are considered working class (choice C is wrong). Lower class: range from no income to $18,000; 27 percent of the population are considered lower class (choice D is wrong).

According to Noam Chomsky, children learn all their vocabulary: A) solely from universal grammar. B) from a complex interaction of genetic and environmental factors. C) solely from environmental factors such as parents and teachers reading to them from an early age. D) from a genetic predisposition to good reading and vocabulary skills.

B. Noam Chomsky strongly believed that a complex interaction of genetic and environmental factors were involved in building children's vocabularies (choice B is correct), although the ability to learn language itself was innate (an idea that was eventually referred to as "universal grammar," a feature unique to humans that allowed them to develop language skills, but is not predictive of vocabulary; choice A is wrong). While Chomsky believed that environmental factors such as good schooling—good teachers and good educational programs—were important in developing children's vocabularies, he also believed that genetic factors played a role in vocabulary development and growth, since not all children in a class had equal vocabulary development (choice C is wrong). Conversely, Chomsky believed that genetics alone was insufficient in accounting for a child's vocabulary strength (choice D is wrong).

The way in which children appear to be acquiring and practicing language skills through the methods described in the research above suggest they are in what stage of language acquisition? (*i.e. by observing and imitating the parents/ engages children reading actively) A) Egocentric speech B) Social speech C) Inner speech D) Private speech

B. Social speech describes the first stage of speech in which children learn by imitating others and by learning to interact with others (choice B is correct). Egocentric speech is the next phase, where children begin to talk to themselves and experiment with language usage (choice A is wrong). Inner speech is the third stage, in which children internalize language development and have an inner sense of grammar and syntax, so that the child no longer needs to practice them aloud (choice C is wrong). Private speech is another term for egocentric speech, the second phase in which children are practicing their language skills aloud but to themselves (choice D is wrong).

Research indicates that professional burnout can result in thoughts of suicide. Suicidal ideation is most often associated with a psychological disorder in which broad category? A) Anxiety disorders B) Mood disorders C) Personality disorders D) Psychotic disorders

B. Thoughts of death or suicide are often associated with major depressive disorder, an example of a mood disorder (choice B is correct). Mood disorders are a group of diagnoses involving disturbances in mood or affect; these disorders describe a longitudinal emotional state. Anxiety disorders are a group of diagnoses involving excessive worries and include generalized anxiety disorder, panic disorder, and obsessive-compulsive disorder (choice A is wrong). Research does not show a significant association between these disorders and suicidal ideation, with the exception of post-traumatic stress disorder. Personality disorders are a group of diagnoses involving enduring maladaptive patterns of behavior and cognition and include antisocial personality disorder, avoidant personality disorder, and borderline personality disorder (choice C is wrong). Psychotic disorders are a group of diagnoses involving a loss of contact with reality and include schizophrenia and delusional disorder (choice D is wrong).

Which of the following is NOT a known environmental risk factor for developing obesity in individuals below the poverty line in the US? A) Limited access to healthcare B) Presence of "food deserts" C) Genetic predisposition D) Lack of green space for exercise or other physical activity

C Genetic predisposition to obesity is not an environmental risk factor for developing obesity, it's a biological risk factor (choice C is correct). For those living below the poverty line in the US, all of the following constitute environmental risk factors contributing to the development of obesity: limited access to healthcare (choice A is wrong), limited access to healthy food (choice B is wrong), and a lack of green space to exercise (choice D is wrong).

Hate crimes committed against immigrants are a consequential example of: A) homophobia. B) xenophobia. C) a social phobia. D) a specific phobia.

B. Xenophobia is defined as the fear of that perceived to be foreign, such as the fear of cultural outsiders (choice B is correct); xenophobic attitudes are one of the negative reactions to immigration, whether due to genuine fear or other economic, political, or social factors, such as increased competition for jobs. Homophobia is a bit more complex and defined as the wide range of negative thoughts about and attitudes and subsequent actions toward non-heterosexual individuals (choice A is wrong); homophobic attitudes are one of the consequences of heterosexism. A social phobia is defined as the fear of potential embarrassment or humiliation in public situations (choice C is wrong). For example, the fear of using a public restroom is a common example of a social phobia that influences a person's abilities to perform routine activities. A specific phobia is defined as the fear of a specific object or situation (choice D is wrong). These are classified into four categories depending on the specific triggers: animal, medical, natural environmental and situational phobias. For example, arachnophobia is a common specific animal phobia of spiders

Where is Broca's Area affected in the brain? Wernicke's area?

Broca's Area (speech production)--> left frontal lobe Wernicke's Area (language comprehension)--> left temporal and part of parietal lobe *language is associated with left hemisphere

Broca's Aphasia vs. Wernicke's Aphasia

Broca's aphasia = difficulty in producing speech Wernicke's aphasia (i.e. receptive aphasia) = speech is preserved but some words are substituted for others or are used incorrectly; utterances may seem confusing or nonsensical

Broca's Area

Broca's area is the part of the brain located in the frontal lobe that is responsible for the construction of language

Heather was recently diagnosed with anorexia nervosa, and she has experienced continued weight loss and severe stomach pain. According to the Learning Theory, for Heather, her physical symptoms would be considered: I. positive reinforcement II. extinction III. punishment A. I only B. III only C. I and III only D. I, II, and III

C According to Learning Theory, positive reinforcement is an operant conditioning term that refers to a favorable consequence that reinforces a particular behavior In Heather's case, the physical symptom of weight loss is considered a reward for her eating behavior (because she has AN and weight loss is desirable for her) and thus this is a positive reinforcement Extinction = process where a behavior in question ceases due to absence of favorable consequence (Heather does not stop her behavior as mentioned in the question, so is eliminated) Punishment refers to a negative consequence experienced as a result of the behavior (i.e. her stomach pains are punishment for her eating behaviors)

An individual goes to the clinic and describes problems with voluntary motor function. He tells the doctor that he is experiencing tremors and weakness that have led to a loss of function in his ankle. After running tests, the doctor cannot find any medical reason for the symptoms which obviously cause the man significant distress and impairment in his occupation (which requires him to stand all day). The individual is referred to a psychiatrist, who shows that the symptoms lack internal consistency by having the individual perform an ankle plantar flexion test both sitting (where he displayed weakness) and then asked the individual to stand on his toes. Having ruled out all other diagnosis, the psychiatrist diagnosed him with what somatic disorder? A. Hypochondriasis B. illness anxiety disorder C. Conversion disorder D. Somatic symptom disorder

C Somatic symptom and related disorders, is a new category in the DSM-5 consisting of disorders described as somatoform disorders in the DSM-IV-TR Hypochondriasis diagnosis was not included in the DSM-5, with most individuals being reclassified as having either somatic symptom disorder or illness anxiety disorder. --> Obsession with the idea of having a serious but undiagnosed medical condition Individuals diagnosed with illness anxiety disorder are often more concerned with illness or the idea of being ill and often lack or have minimal somatic symptoms. In somatic symptom disorder, the focus of the individual is on the high level of distress or anxiety that is disproportionate to the seriousness of the symptoms being exhibited. The DSM-5 criteria for conversion disorder states the individual must exhibit at least one symptom of altered voluntary motor or sensory function that shows internal inconsistency, causes distress or impairment, and cannot be explained by another mental or medical disorder. The ICD-10-CM categorizes by symptom type, with weakness (or paralysis) and abnormal movement (tremor) listed as specific symptom types. **Somatization is the psychological mechanism whereby psychological distress is expressed in the form of physical symptoms. The psychological distress in somatization is most commonly caused by a mood disorder that threatens mental stability. **Conversion disorder occurs when the somatic presentation involves any aspect of the central nervous system over which voluntary control is exercised. Conversion reactions represent fixed ideas about neurologic malfunction that are consciously enacted, resulting in psychogenic neurologic deficits.

Which of these cohorts is most likely to have experienced at least one major depressive episode over the last 12 months? A. People between 26 and 49 years of age B. People over the age of 50 C. Women D. Men

C The period prevalence is the proportion of a specific population exhibiting a certain characteristic, assessed over a specific period of time. People over the age of 50 have the lowest period prevalence of any age group. Men have a much lower period prevalence than women. Although people between the ages of 26 and 49 experience more than people over the age of 50, they are less likely to experience a major depressive episode than women as a group.

Unlike the DSM-IV-TR, the DSM-5 makes a distinction between paraphilias and paraphilic disorders by stating that "a paraphilia is a necessary but not a sufficient condition for having a paraphilic disorder". What does this difference in approach toward the description of paraphilic disorders imply? A. The APA believes that individuals engaging in nonnormative sexual practices have paraphilic disorders. B. The APA has changed the description for paraphilia to allow it to cover more forms of nonnormative behavior. Labeling these behaviors as psychopathological allows more people with paraphilic disorder to obtain help. C. The APA did not change the description of paraphilic disorders, but rather emphasized that nonnormative behavior must cause distress or impairment, before labeling the behavior as psychopathological. D. The APA is making a distinction between normative and nonnormative behavior, labeling nonnormative behavior as psychopathological.

C. The DSM-5 describes paraphilia as any intense and persistent sexual interest other than genital stimulation or fondling in phenotypically normal, physically mature, and consenting human partners. Paraphilias include sexual sadism (inflicting humiliation, bondage, or suffering), masochism (being humiliated, bound, or suffering), transvestic (sexually arousing cross-dressing, in addition to voyeurism (spying on others) and frotteurism (touching or rubbing genitals against a nonconsenting individual), and pedophilia (sexual focus towards children). The APA did not change which behaviors are labeled as normative or nonnormative paraphilias, but instead focused on whether the paraphilia causes distress or impairment to the individual or others before labeling the behavior as psychopathological.

If you become violently ill after eating at a new restaurant, and feel nauseated whenever you pass that restaurant in the future, this feeling of nausea is an example of: A) operant conditioning. B) negative reinforcement. C) classical conditioning. D) unconditioned response.

C. Becoming violently ill after eating something new is a prime example of classical conditioning, demonstrating that we are biologically wired to learn taste aversions to foods that make us ill. Becoming ill is the unconditioned response, the food is the unconditioned stimulus, the new restaurant is the conditioned stimulus, and the nausea experienced upon passing that particular restaurant is the conditioned response (choice C is correct; choice D is wrong). This example does not accurately represent operant conditioning, as there was no immediate goal of behavior modification (choice A is wrong). Getting ill would actually best be viewed as a punishment, since nausea is a negative stimulus, whereas negative reinforcement is defined as removing an aversive stimulus to reinforce a positive behavior (choice B is wrong).

If one woman in a small, isolated Korean village fell ill with shin-byung and her close friend then also began experiencing similar symptoms, claiming she too was being pursued by spirits, which of the following might explain this occurrence in the friend? A) Compliance followed by obedience B) Conformity followed by compliance C) Conformity followed by private acceptance D) Obedience followed by conformity

C. Conformity, obedience and compliance are related but distinct terms. Conformity is changing one's thoughts/behavior in order to fit in with the norms of a particular social group, most typically a group that has a certain level of social importance. Obedience is changing one's thoughts/behavior in response to the direct command or order of a person who is in authority or of higher social status. Compliance means changing one's behavior in response to a request from another person who is of equal or lower social status (thus differing from obedience). Private acceptance is an attitude change that can occur in a person due to the social influence of others. Thus, it is most likely that the friend first conformed to the behavior of the woman initially affected by shin-byung symptoms, then privately accepted the possibility of spirit possession after modifying her behavior (choice C is correct). One could comply with a request from a peer and then obey a command from a leader but that would not explain the situation described (choice A is wrong). In addition, changing behavior in accordance to a group norm followed by responding to a request from a peer would not fit the described situation (choice B is wrong). Finally, following the orders of an authority figure and then the group norms of a desired social group would not explain the shin-byung symptoms in this example (choice D is wrong).

Knowledge of specific olfactory thresholds for various substances would be most useful to which of the following clinicians? A) An eating disorders counselor trying to rule out a diagnosis of anorexia nervosa B) A psychologist trying to determine if a patient has a cocaine addiction C) A psychiatrist assessing for positive symptoms of schizophrenia D) A therapist treating elderly individuals with delirium

C. Positive symptoms of schizophrenia include delusions and hallucinations. While hallucinations typically occur in either the visual or auditory senses, they can occur in any of the five senses, including the olfactory sense. Thus, by knowing the detection threshold for various substances, a clinician could determine if a patient's claim that he/she smelled a particular substance was a function of hallucinations or an accurate olfactory detection (choice C is correct). While cocaine can be snorted, the ability of a patient to actually detect the odor of cocaine would be irrelevant to their addiction (choice B is wrong). Olfactory processes are not associated with anorexia nervosa or delirium (choices A and D are wrong). *Olfactory (smell)

Effective psychological treatments for anxiety disorders share which common treatment element? A) Cognitive restructuring B) Behavioral activation C) Exposure D) Relaxation

C. Effective psychological treatments for anxiety disorders all share the common treatment element of exposure to lessen anxiety (choice C is correct). By allowing the individual to face what he or she deems too terrifying, they are confronting the source of their fear, facilitating the extinction of a classically conditioned response, and/or allowing their body to habituate to the feelings of fear or anxiety, and thus lessen anxiety. Cognitive restructuring can be one type of treatment element for certain anxiety disorders, such as restructuring maladaptive cognitions in Generalized Anxiety Disorder, but cognitive restructuring is not helpful in breaking the cycle of interoceptive conditioning in Panic Disorder or controlling urges in Obsessive Compulsive Disorder (choice A is wrong). Behavioral Activation is an empirically supported treatment of depression, not anxiety disorders, that uses Skinnerian psychological models of behavior change to improve mood (choice B is wrong). Relaxation training is a prominent behavioral technique for the treatment of Generalized Anxiety Disorder, but is not as helpful for other anxiety disorders, in which the individual may need to confront and remedy the source of the fear/anxiety (choice D is wrong).

The idea that foreign-born students might be negatively affected by native-born students' cultural norms and values can best be explained by: A) cultural relativism. B) prejudice. C) ethnocentrism. D) discrimination.

C. Ethnocentrism is the tendency to believe that one's own ethnic or cultural group is most important, and all other groups are measured against one's own. Ethnocentric thinking typically makes false or negative assumptions about a minority ethnic or cultural group (choice C is correct). Cultural relativism is seen as the opposite of ethnocentrism, as cultural relativism is the principle of viewing aspects of a different culture from the viewpoint of that culture, rather than from one's own viewpoint (choice A is wrong). Prejudice is a negative and often unjustified attitude or stereotypical beliefs towards an individual or a group, and it is not specific to race or culture (choice B is wrong). Discrimination is actual behavior or actions, typically negative, towards an individual or group (choice D is wrong).

When we are stressed, the immune system mobilizes specialized cells that work to protect the body from diseases. What cells are mobilized in this process? I. Antigens II. Lymphocytes III. Immunogens A) I only B) III only C) II only D) I and III only

C. Item I is false: Any foreign substance that, when introduced to the body, carries the potential to stimulate the immune system is considered to be an antigen; the body produces antibodies to protect against antigens, but antigens are foreign to the body (choice A and D can be eliminated). Item II is true: Lymphocytes are part of the cellular barrier protecting the immune system. Lymphocytes, such as B cells and T cells, are specialized leukocytes (or white blood cells) and have specific tasks in protecting the body from attack. Item III is false: An antigen that stimulates the lymphocytes to produce antibodies or to attack the antigen directly is called an immunogen. Antigens and immunogens are designed to attack the immune system, not protect it (choice D can be eliminated; choice C is correct).

There are multiple sociological perspectives on deviance. Which of the theorists below are expected to consider the following questions about non-normative behaviors? I. Differential association theorists; "How can people resist deviance? II. Labeling theorists; "Who defines deviance?" III. Structural strain theorists; "How do norms affect deviance?" A) III only B) I and II only C) II and III only D) I, II, and III

C. Item I is false: Differential association argues that deviant behaviors are learned through interactions between individuals and their communities. These theorists contend that people become deviant when their contacts have an excess of definitions favorable to deviant behaviors. "How can people resist deviance?" is therefore not an expected question (choices B and D can be eliminated). The main criticism of differential association is that it reduces individuals to their environment. Because it states that those in deviant communities learn to be deviant themselves, the possibility of resistance is not considered. Item II is true: Labeling theory argues that deviant behaviors are seen as the result of social processes of labeling. "Who defines deviance" is therefore an expected question (choice A can be eliminated). Labeling theorists do indeed address this concern; for example, these theorists are interested in the mechanisms through which power contributes to the construction of deviance (e.g., agents of social control). Item III is true: Structural strain theory argues that deviant behaviors are the result of tension between the accepted social goals and the institutionalized means available to achieve these goals. "How do norms affect deviance?" is therefore an expected question (choice C is correct).

A student received a series of poor grades on essay assignments that required her to write on an unfamiliar and difficult topic. Later in the school year, she was given writing assignments on a more familiar topic, but she put in very little effort because she was convinced that she was a poor writer. Her experience is closest to which of the following psychological theories? A) Social learning B) Social loafing C) Learned helplessness D) Self-serving bias

C. Learned helplessness occurs when repeated failure discourages further effort, even when circumstances change (choice C is correct). Social learning theory posits that we learn new behaviors through imitation (choice A is wrong). Social loafing theory states that when we are members of a group, we work below our individual potential (choice B is wrong). The self-serving bias attributes success to internal factors and failure to external ones; there is no evidence that the student is attempting to attribute blame for her poor performance to external factors, and she is convinced that her failure is due to an internal cause, so the self-serving bias does not explain the scenario in the question stem (choice D is wrong).

Which of the following is NOT a type of parasomnia? A) Somnambulism B) Bruxism C) Night eating syndrome D) Confusion arousals

C. Parasomnia encompasses an array of conditions that arise before or during sleep. Although night eating syndrome occurs at night, it does not actually disrupt sleep. After all, it occurs before the individual has fallen asleep (choice C is correct). Somnambulism, or sleepwalking, occurs during, and therefore disrupts, sleep (choice A is wrong). Bruxism, which is more commonly called grinding (of the teeth), generally occurs at night and can disrupt sleep patterns (choice B is wrong). Confusion arousals, by which the individual feels confused or lost while waking up or just after awaking, are considered a type of parasomnia as well (choice D is wrong).

Which of the following characteristics primarily differentiate between schizophrenic behavior and schizoid affect? A) Schizophrenics hear voices and schizoids have delusions. B) Schizophrenics have delusions and schizoids have thought disorders. C) Schizophrenics show psychotic behaviors and schizoids withdraw from interactions with others. D) Schizophrenics show flat affect and schizoids have bursts of wild behavior.

C. Schizophrenia is classified as a brain disorder that may include any combination of the following symptoms: positive symptoms (familiarly known as psychotic symptoms such as delusions, hallucinations, hearing voices, thought disorders, or movement disorders) and negative symptoms (including a flattened affect, a loss of pleasure in everyday life or previously pleasurable activities, little interest in beginning or sticking with activities, or in speaking with others). Cognitive symptoms include poor executive functioning (e.g., decision-making), or trouble focusing or remembering (therefore, choices A, B, C, and D all accurately describe schizophrenia symptoms). "Schizoid" refers to a personality disorder, a different class of diagnosis, and is characterized primarily by an unwillingness to interact with others. Those with schizoid personality disorders may seem to be aloof or loners, and may or may not enjoy that state of being (choice C is correct). There are no additional behavioral components to schizoid personality disorder other than the withdrawal from social interaction. Schizoids do not have delusions (choice A is wrong), do not have thought disorders (choice B is wrong), and do not demonstrate bursts of wild behavior (choice D is wrong).

What famous social psychology experiment was designed to study in-group favoritism? A) Solomon Asch's conformity experiments B) Stanley Milgram's shock experiments C) The Robbers Cave experiment D) The Tuskegee syphilis experiment

C. The Robber's Cave experiment showed that even arbitrary group distinctions (camp teams) can cause bitter rivalry and discrimination, thus demonstrating in-group/out-group biases (choice C is correct). Solomon Asch's conformity experiments looked at the likelihood of conformity in a group setting (choice A is wrong) and Stanley Milgram's shock experiments analyzed obedience and the power of authority figures (choice B is wrong). The Tuskegee syphilis experiment was an extremely unethical medical experiment (not social psychology experiment) that demonstrated a fair amount of race bias on the part of the experimenters toward the participants, but was not attempting to study in-group/out-group bias (choice D is wrong). --> The 40-year study was controversial for reasons related to ethical standards because researchers knowingly failed to treat patients appropriately after the 1940s validation of penicillin as an effective cure for the disease they were studying. Now studies require informed consent [3] communication of diagnosis, and accurate reporting of test results.

Which personality disorder describes someone who has a history of unstable relationships, a diffuse and unreliable sense of identity, chronic feelings of emptiness, and a history of suicidal thoughts and gestures? A) Schizotypal personality disorder B) Dependent personality disorder C) Borderline personality disorder D) Antisocial personality disorder

C. The personality disorder characterized by unstable identity and interpersonal relationships, as well as chronic feelings of emptiness and a history of suicidal thoughts and gestures is borderline personality disorder (choice C is correct). Schizotypal personality disorder is characterized by acute discomfort in interpersonal relationships, to the point that people with this disorder rarely have friends outside of immediate family members, and odd eccentric behavior. People with this disorder also frequently have notable cognitive or perceptual disturbances (choice A is wrong). Dependent personality disorder is characterized by a difficulty with everyday decisions, a need for advice and reassurance from others, a difficulty expressing disagreement with others out of a fear of loss of approval or support, feelings of helplessness or discomfort when alone, and an unrealistic preoccupation with fears of being abandoned. Additionally, people with this disorder have difficulty initiating projects, and go to excessive lengths to obtain nurturance or support from others (choice B is wrong). Antisocial personality disorder is characterized by a pervasive pattern of unconcern for and violation of the rights of others, which can be see through repetitive rule-breaking or failure to conform to social norms, failure to plan ahead, impulsivity, irritability and/or aggressiveness, irresponsibility, and a lack of remorse. People with this disorder do not have an unstable sense of identity, nor do they generally regularly contemplate suicide without extreme provocation (choice D is wrong).

Many forces influence how people relate to one another, become socialized beings, and learn to develop relationships. Which of the following is NOT one of the agents of socialization? A) Family B) School C) Travel D) Television

C. While travel can be highly educational, introducing people to other cultures and ways of life, it is not specifically an agent of socialization, helping people to learn how to relate, as one can travel alone and avoid contact with others (choice C is correct). Family is one of the first and primary sources of socialization in which a person learns how to relate to others—one's parents, siblings, and extended family, depending on family structure. The lessons learned here often form the bases for many future relationship choices (choice A is wrong). School is another primary source of socialization, as teachers try to educate children about the values of the specific culture and the important skills needed to survive in that culture (choice B is wrong). Television, along with other forms of mass media, provide people with a glimpse of how to relate to the larger world, introducing people to other lifestyles as well as popular trends and ideas (choice D is wrong).

Tony has recently come out to his friends as transgender; despite his female anatomical features, he wants to live and function socially as a man. Tony's friends were very supportive, but it bothers Tony that his friends make such a big deal about his gender transition when introducing him to new acquaintances or just hanging out. Based on this information, which of the following best describes Tony's transgender identity? A) It is his ascribed status. B) It is his achieved status. C) It is his master status. D) It is his status symbol.

C. A master status is when one role (in this case, being transgender) becomes more prominent than the other social roles that comprise one's identity (choice C is correct). Tony has become "the transgender friend." If being transgender was not his master status, Tony might have been known as "the medical student friend who loves to travel and volunteer at the animal shelter and happens to be transgender." An ascribed status is a role that is assigned to someone by society despite that person's efforts to take on another role (choice A is wrong). An achieved status is a role that a person has taken on through their own efforts. Although Tony has taken on a role as a transgender person due to his own efforts, an achieved status does not become all-encompassing as a master status would (choice B is wrong). A status symbol is a component of impression management. It is generally an object that is displayed in order to effect a certain image. This is unrelated to the information given in the question stem (choice D is wrong).

If you have a high sensitivity to a smell, do you have a high or low detection threshold?

Low detection threshold Because you would sense it faster

Conflict theorists are concerned with stratification in societies. The argument least consistent with this perspective is which of the following? A) Stratification is the result of competition between different social classes. B) Stratification is the result of the exploitation of subordinate groups. C) Stratification is the result of the need for the maintenance of social order. D) Stratification is the result of capitalist motivation to accumulate wealth.

C. The conflict perspective argues that social structures, including structured social stratification, reflect the competition for limited resources (choice A is consistent and can be eliminated). These structural inequalities happen because those with the most power (the dominant groups) maintain their positions of power through the suppression of those with the least power (the subordinate groups; choice B is consistent and can be eliminated). The traditional conflict theorists found this to be of particular interest in capitalist societies; Marx, for example, argued that capitalism produced internal tensions between those who controlled the means of production and those who labored the production (choice D is consistent and can be eliminated). However, the argument that stratification contributes to social order is not expected to be made from the conflict perspective; this is better described as a functionalist argument (choice C is not consistent and is the correct answer). For example, Marx argued that the tensions associated with capitalism would lead to its self-destruction, suggesting that he did not find pronounced stratification to contribute to social order.

In Erikson's psychosocial stages of development, which stage occurs when it involves helping the next generation and contributing to others and society?

Generativity vs Stagnation **Generativity is characterized by activities that are productive or that transcend one's own mortality. The transmission of knowledge or wisdom to the younger generation is one illustration of this kind of generativity

Access to and utilization of preventative care is often evaluated through a community's use of prophylactics, defined as medical or public health procedures that prevent disease. Immunization is a common and widespread example of prophylaxis. Additional examples of prophylaxis would be expected to include all of the following, EXCEPT: A) birth control methods, including condoms. B) regular moderate physical activity. C) cesarean section (c-section) delivery. D) fluoride therapy and tooth cleaning.

C. The question describes prophylaxis as a method of preventative care. Birth control can be used as a preventative measure, often to prevent pregnancy (choice A is an example of prophylaxis and can be eliminated). Furthermore, considering the example provided, condoms can also be used to prevent sexually transmitted disease. Physical activity has numerous health benefits, including the ability to reduce one's risks for diseases such as heart diseases, muscular and skeletal diseases, and even some forms of cancer (choice B is an example of prophylaxis and can be eliminated). Cesarean section, although a common procedure, is a surgical method; it is performed when physicians deem it a safer method of delivery, but is generally not used preventatively, but rather as a response to complications, especially in the first instance (choice C is not an example of prophylaxis and is the correct answer). Fluoride therapy and tooth cleaning are dental forms of prevention, helping to prevent dental disease, such as gingivitis, which could have a negative effect on overall health (choice D is an example of prophylaxis and can be eliminated).

How might Anne Treisman's model account for the results in this research? A) Words and circles enter the central executive, go through the phonological loop, then the visuospatial sketchpad, and the episodic buffer, and then into working memory. B) Words and circles enter a sensory store, go through a selective filter, and hit a bottleneck; only the words go through to higher level processing and then working memory. C) Words and circles enter a sensory store, go through an attenuating filter, hit a bottleneck, where the "volume turns up" on the words and down on the circles, and words get stored in working memory. D) Words and circles enter a sensory store, go through a phonological filter, hit a bottleneck, and both words and circles reach working memory.

C. Treisman developed the attenuation model for selective attention, which would explain the research above as the words and circles having entered the sensory store, gone through an attenuating filter. At that point, it hits a bottleneck, where the "volume turns up" on the words and down on the circles, so that the words can be stored in working memory, and then recalled (choice C is correct). Words and circles entering a central executive into a phonological loop, a visuospatial sketchpad, and episodic buffer before arriving at memory describes Baddeley's Model of Working Memory (choice A is wrong). Words and circles going through a selective filter describes the selective filter theory of selective attention (choice B is wrong). A pathway in which words and circles enter a sensory store, proceed to a phonological filter and then reach working memory is actually an amalgam of Baddeley's model and the selective filter model and does not exist (choice D is wrong).

A right-handed patient is admitted to the hospital for a traumatic brain injury. If his speech is fluent but nonsensical, he most likely received a blow to the: A) right parietal lobe. B) right temporal lobe. C) left frontal lobe. D) left temporal lobe.

D. Fluent but nonsensical speech describes Wernicke's aphasia, in which the patient may have trouble processing language but can produce words in a faux-grammatical fashion. It stems from damage to Wernicke's area and the surrounding tissue, specifically the left temporal lobe and part of the left parietal lobe (choice D is correct). Language is lateralized in the left hemisphere, particularly in right-handed individuals, so this deficit would not have been caused by a blow to the right hemisphere (choices A and B are wrong). Damage to the left frontal lobe might have instead damaged Broca's area, which would have left the patient with the ability to comprehend language but not produce it (choice C is wrong).

Linguist Noam Chomsky proposed all of the following, EXCEPT: A) all children are born with an innate ability to learn language. B) humans are capable of learning language through mere exposure to it. C) there is period during development that is sensitive to learning language. D) children learn language through a series of positive reinforcements.

D. B.F. Skinner's behaviorist model of language development proposed that children learn language through a series of positive reinforcements Chomsky proposed something entirely contrary to this theory (choice D is correct). Noam Chomsky proposed that all children are born with an innate ability to learn language (choice A is wrong) during the sensitive developmental years during early childhood (choice C is wrong). Chomsky also proposed that children learn language through mere exposure to it, and do not need to be actively taught language (choice B is wrong).

Jax, a gay teenager, was kicked out of his parents' house when he came out as gay. Now he lives with a community of other teenagers in a youth center and he considers this community to be his family. Which of the following specific types of groups is this community for Jax? I. An in-group II. A primary group III. A reference group A) I only B) I and II only C) II and III only D) I, II, and III

D. Item I is true: an in-group is a group that an individual belongs to and feels him/herself to be a member of (choice C can be eliminated). Item II is true: a primary group serves expressive functions, such as meeting emotional needs. A family is an example of a primary group and Jax considers his teenage friends to be his family (choice A can be eliminated). Item III is true: a reference group is a group that one might compare oneself to based on the fact that all members share a common identity. Jax can compare himself to his teenage friends because they are all in a similar situation—being young and living primarily in shelters (choice D is correct; choice B can be eliminated).

Normative social influence describes: A) the process by which humans learn the norms of their society. B) a process whereby an individual conforms their behavior to match everyone else's behavior because he or she wants to do the right thing and feels that everyone else knows something he or she doesn't know. C) the likelihood that an individual's behavior will be considered "normal" in any given situation. D) a situation where a person's behavior conforms or changes because she or he wants to be liked by others.

D. Normative social influence describes when someone changes her or his behavior (conforms) because she or he has a desire for the approval of others and wants to avoid rejection (choice D is correct). Socialization is the term generally used to describe the process by which humans learn the norms of their society (choice A is wrong). Informational social influence describes when an individual's behavior conforms or changes because he or she wants to do the "right thing" (choice B is wrong). The likelihood that an individual's behavior will be considered "normal" in any given situation is not described by normative social influence (choice C is wrong).

The finding that older people retrieve autobiographical memories better than other types of memories would be best accounted for by: A) the retrieval structure principle. B) self-serving bias. C) fluid intelligence. D) the self-reference effect.

D. People tend to remember things about themselves better than other types of information; this is known as the self-reference effect (choice D is correct). The retrieval structure principle refers to a strategy of initial encoding whereby information is stored in a manner that allows for greatest accessibility in the future, but it is not limited to autobiographical information (choice A is wrong). Self-serving bias is an observation of social psychologists that describes people's tendency to view themselves in an unrealistically favorable manner, but it is not directly associated with memory retrieval (choice B is wrong). The ability to think logically and apply information to solve novel problems is termed fluid intelligence, which is in contrast to previously stored information called crystallized intelligence (choice C is wrong).

Robert Merton proposed the concept of unintended consequences, defined as unintended outcomes of purposeful social action. This suggests the importance of routinely evaluating models of social intervention, such as global health initiatives, for unintended consequences. For example, coercive vaccination caused resistance among individuals and communities during smallpox eradication efforts in India. In this case, the consequential drug resistance is best described as which of the following? A) A manifest function B) A latent function C) A manifest dysfunction D) A latent dysfunction

D. Robert Merton is the theorist credited for the distinction between manifest and latent functions and dysfunctions. In the global health situation described, the reported drug resistance among individuals and communities in India is an example of a latent consequence of social intervention (choices A and C are wrong). Latent functions and dysfunctions are the unrecognized, unintended consequences of social patterns; manifest functions and dysfunctions are the recognized, intended consequences of social patterns. Furthermore, this can be described as social dysfunction, rather than a function, because it is an undesirable consequence that can reduce social stability (choice B is wrong and choice D is correct). Thus, the two-by-two theory could be applied to choose the correct answer here.

Gentrification is best described as an example of which form of demographic change? A) Urban blight B) Urban decline C) Urban growth D) Urban renewal

D. The process through which urban communities are redefined, called gentrification, is most often the result of urban renewal efforts (choice D is correct). This calls for the redevelopment of urban areas, and gentrification, in particular, suggests a shift in the communities through economic developments that attract wealthier businesses and therefore residents. This is similar to the concept of urban growth, but gentrification is specific to the revitalization of existing urban centers; urban growth is a more ambiguous concept and most often studies of urban growth are concerned with the initial growth of cities as a result of historic industrialization (choice C is wrong). However, the passage states that gentrification is the opposite of urban decline as it concerns the process through which people re-enter cities (choice B is wrong). Urban blight is one of the results of urban decline in which cities that were once functioning rot to a state of disrepair; this is exemplified through "ruin porn" that features desolate landscapes (choice A is wrong).

Brain imaging studies on willing deceit have shown that this leads to an alternate cognitive process when compared to truth telling, which appears to be the default process of the human brain. In suppressing or inhibiting the truth, and often responding to subsequent stress, the brain becomes more active overall. Multiple areas are involved in the process, which are expected to include which of the following regions? I. The frontal lobes II. The limbic system III. The temporal lobes A) II only B) I and III only C) II and III only D) I, II, and III

D. The two main brain structures expected to be activated as a result of suppressing or inhibiting the truth are the cerebral cortex and the limbic system. The general functions of the cerebral cortex include perception, skeletal muscle movement, memory, attention, thought, language, and consciousness; the general functions of the limbic system include emotion, memory, and learning. The regions of the cerebral cortex expected to be involved in the process described in the question stem are the frontal lobes and the temporal lobes. Item I is true: The frontal lobes are involved in complex reasoning skills and problem solving, both of which could occur in the process of suppressing or inhibiting the truth (choices A and C can be eliminated). Item II is true: The limbic system is involved in both memory retrieval, which allows the brain to compare the mental images (i.e., the truth versus the lie), and the resulting emotions related to deception (e.g., anxiety; choice B can be eliminated and choice D is the correct answer). Item III is true: The temporal lobes are involved in short-term memory, language comprehension, and emotion; suppressing or inhibiting the truth consciously suggests memory retrieval.

Behavioral theorists explain which personality disorder as the result of having been rewarded for loyalty and punished for acts of independence? A) Histrionic personality disorder B) Obsessive-compulsive personality disorder C) Avoidant personality disorder D) Dependent personality disorder

D. With dependent personality disorder, individuals demonstrate patterns of clinging and obedience and fear separation, often insisting that they need to be taken care of. Behavioral theorists believe that individuals with dependent personality disorder developed their disorder because of how they were treated by their parents when growing up; they were unintentionally rewarded for being clingy ("loyal") and were punished when they acted on their own (choice D is correct). Histrionic personality disorder, a disorder characterized by excessive emotionality and attention-seeking, is more often explained in terms of children having learned to behave dramatically to get attention from distant parents (choice A is wrong). Obsessive-compulsive personality disorder may arise from struggles with parents over independence and control but is not thought to involve reward and punishment (choice B is wrong). Behavioral theorists posit that individuals with avoidant personality disorder fail to develop normal social skills and come to expect rejection (choice C is wrong).

According to ancient, traditional Judaism, the first-born son must be redeemed through pidyon haben. This means that he will not be required to serve as a priest as outlined in holy texts. Accordingly, thirty days after the child is born (unless the date falls on Shabbat), his parents must redeem him by paying the modern equivalent of five silver shekels. Pidyon haben is an example of a: A) belief. B) value. C) norm. D) ritual.

D. A ritual is an action or ceremony that is performed consistently under prescribed circumstances. Pidyon haben takes place in the Jewish faith when a first son is born (prescribed circumstances); the exchange of money and the time the event occurs is also consistent from family to family. Pidyon haben, therefore, is a form of ritual (choice D is correct). A belief is something one assumes about the world. Many Jews therefore subscribe to the belief that Pidyon haben is necessary; however, the event itself is not a belief (choice A is wrong). Values often derive from beliefs and correspond to ideas and concepts individuals deem important. Honesty or integrity, for example, might be considered values by a population (choice B is wrong). A norm is a standard maintained by a society about how individuals should behave in particular situations. For instance, in a synagogue, individuals would behave in accordance with established norms (choice C is wrong).

It has been proposed that there may be variations in the OXTR gene for individuals with and without a specific psychological disorder. In particular, researchers cite the OXTR gene's role in a variety of social behaviors, including trust, social comprehension, and attachment as being evidence that it may be different between people with and without this disorder. It is likely that researchers think that variation in the OXTR gene contributes most to which of the following disorders? A) Schizophrenia B) Obsessive-compulsive disorder C) Attention deficit/hyperactivity disorder D) Autism spectrum disorders

D. Although research does suggest that OXTR gene may be associated with general psychopathology, autism spectrum disorders are most characterized by deficits in social behaviors such as trust, social comprehension, and attachment. Research has suggested that autistic individuals may be more likely to exhibit an A allele (choice D is correct). Schizophrenia is often characterized by delusional thinking, hallucinations, and disorganized behavior, but social deficits are often only a byproduct of other psychotic symptoms (choice A is wrong). Obsessive-compulsive disorder is characterized by persistent and obsessive thoughts and repetitive behaviors, but is also not strongly linked to social deficits (choice B is wrong). Attention deficit/hyperactivity disorder (ADHD) is characterized by problems with maintaining attention and controlling impulsive behaviors, but is only tangentially linked to social deficits (choice C is wrong).

Assuming that Korean culture is androcentric, which of the following statements is most plausible? A) In Korea, women are considered strong and are therefore less susceptible to shin-byung. B) A woman given this diagnosis of shin-byung in the United States would receive the same social prestige as in Korea. C) Korean women have little motivation to seek out attention and prestige by claiming the symptoms of shin-byung. D) A Korean man who seeks help for shin-byung symptoms would most likely receive a different diagnosis.

D. An androcentric culture is one that is male oriented; the male is considered the norm and is consistently preferred across the culture. As a result of this, anything that is related to females and the female experience is relegated to a lower tier of importance. If a male would claim to be experiencing these symptoms, it is unlikely that he would be treated the same as a female and would most likely receive a different diagnosis, particularly because the passage seems to indicate that this occurs only in Korean women (choice D is correct). In an androcentric culture, it is less likely that a women would be seen as strong enough to resist this disorder (choice A is wrong). Another culture, which neither recognizes this culture bound disorder nor rewards women for becoming shaman (such as in the U.S.), is unlikely to provide the same social rewards for women with these symptoms (choice B is wrong). If there are limited avenues for women to gain social status in an androcentric society, there is a greater motivation for women to use unusual means to gain status for themselves (choice C is wrong).

According to Piaget's theory of development, at which stage do children learn that things can be represented through words and images? A) Formal operational stage B) Sensorimotor stage C) Concrete operational stage D) Preoperational thought stage

D. During Piaget's preoperational thought stage (around ages 2 to 7), children learn that things can be represented through symbols, such as words and images; this is an important foundation for the development of more advanced language skills. At this stage they still lack logical reasoning but they begin to understand the world of representational symbols, so it is during the preoperational thought stage where children learn that things can be represented through words and images (choice D is correct). The formal operational stage occurs when a child reaches the age of 12 and above. This is where the child begins to understand complex abstract thought and reasoning, as well as morality (choice A is wrong). The sensorimotor stage occurs during the first two years of life where a child explores his or her world with the five senses and through movement. This is also where they learn object permanence (choice B is wrong). The concrete operation stage is from ages 7 to 12, where the important lesson is conservation—that quantities remain the same even if one changes their shape. This is also the age where children begin to grasp abstract mathematical concepts (choice C is wrong).

What happens in Parkinson's disease?

In Parkinson's disease, neurons in the brain that make dopamine and control muscle movement begin to die.

All of the following biological developmental milestones that change the brain occur during the adolescent years, EXCEPT: A) Cell proliferation (particularly in the limbic system and prefrontal lobes) B) Myelination C) Synaptic pruning (of unused or unnecessary connections) D) Codification of neural networks for basic motor skills

D. Neural network development for motor skills is part of early brain development and is essentially complete for basic motor skills (such as crawling, walking, jumping, writing, etc.) before adolescence (choice D is correct). Cell proliferation, particularly in the prefrontal lobes and in the limbic system, is one of the changes to the brain that occurs during adolescence, as the prefrontal cortex is required for abstract thinking (choice A is wrong). Myelination, the creation of myelin sheaths around neurons in the brain to strengthen connections, is another change during adolescence (choice B is wrong). Synaptic pruning, which clears the brain of unnecessary connections, is the third major developmental change in the brain during adolescence (choice C is wrong).

Later in her interview Subject 103 talked about her experience with smoking. She mentioned that she was interested in quitting, but that stopping smoking made her feel irritable and stressed. Smoking was the only thing that calmed her down, she told the interviewer. In this case, irritability is one sign of: A) psychological dependence. B) biofeedback. C) physiological arousal. D) physical dependence.

D. Physical dependence is evidenced by symptoms of withdrawal that are calmed by further use of the addictive substance. In this case, Subject 103 experiences irritability, but it is calmed by the nicotine she gets from smoking (choice D is correct). Psychological dependence can result from using a drug to cope with painful emotions. Although Subject 103 experiences painful emotions because she does not have a community that accepts her identity, the information in the question stem deals with her physical symptoms of dependence and not with the potential underlying psychological causes (choice A is wrong). Biofeedback is a therapeutic technique in which individuals are given information about their autonomic responses to stressful situations and taught to use this information to control their responses (choice B is wrong). Physiological arousal is an excitation of the body's internal state; it is an aspect of emotional response (choice D is wrong).

Which of the following is NOT a theory that accounts for gender differences? A) Gender Stereotyping B) Gender-Role Preferences C) Gender-Schema Theory D) Sex Drive

D. Sex drive in humans is a function of physiological (e.g., hormonal) and nonphysiological factors, but has nothing to do with gender difference (choice D is correct). Gender stereotyping describes the all-pervasive ideas in the culture that denote that certain types of clothes, comportment, toys or colors, for example, are appropriate for one gender, and others for another (choice A is wrong). Gender role preference is when a child decides early on that despite having the physiological and biological characteristics of one gender, that child would prefer to be a different gender and proceeds to adopt the comportment of that gender role (choice B is wrong). Gender schema theory explains how gender differences are transmitted. The theory incorporates gender stereotypes with biological characteristics (choice C is wrong).

Children learn their roles in the world from their parents and other social influences. Roles and rules vary across families and cultures. For instance, spitting may be deemed offensive in one household but acceptable in another. Specific norms are therefore created by individuals and would not exist in the absence of human beings. This idea that people develop meanings of the world and thereby agree upon values is grounded in the theory of: A) symbolic interactionism. B) functionalism. C) conflict theory. D) social constructionism.

D. Social constructionism is concerned with the ways in which people understand, interpret, and give meaning to the world. It explains how human beings socially construct such aspects of life as roles and rules. Therefore, the same action, such as spitting, can be interpreted differently by different groups of people. The taboo, however, would not exist if humans had not created it in the first place (choice D is correct). Symbolic interactionism studies small scale interactions between and among human beings. It argues that people act based on meanings they have assigned to objects; but such meanings are fluid and subject to change. While symbolic interactionism is very similar to social constructionism, it is important to note that only the former focuses on the way people act (choice A is wrong). Functionalism involves multiple parts of an entire unit or society working together to establish equilibrium (choice B is wrong). Conflict theory refers to the ways in which societies transform over time. Oftentimes, two opposing viewpoints arise; constant competition leads to an ever-changing world. This perspective also focuses on social control and power struggles (choice C is wrong).

The resource model of attention suggests that: A) humans must "shine a spotlight" on the particular resource on which they wish to focus in order to focus full attention. B) humans have the resources to fill in any missing information and thus do not need to attend to everything. C) humans selectively filter out unnecessary information according to their resources. D) humans have a limited amount of resources for attention when performing tasks.

D. The resource model of attention states that humans have a limited amount of resources for attention. Thus, if one wants to multitask, and the attentional needs exceed available resources, these tasks cannot be completed simultaneously (choice D is correct). The spotlight theory of attention is related more to visual attention and refers to the idea of where one focuses attention in one's visual field, as opposed to where one moves his or her eyes (choice A is wrong). That humans fill in missing information is related to the priming effect where if people are "primed" or exposed repeatedly to particular sensory data, they are more likely to notice it. For example, if one is primed to be aware of monsters, he or she may be more likely to fill in the blank with "ster" if presented with "mon----" (choice B is wrong). The idea that humans selectively filter out unnecessary information is part of the Broadbent Filter Model of Selective Attention, which states that sensory information enters a buffer and is then filtered based on various characteristics. Some sensory input decays while other data moves on to short-term and even long-term memory (choice C is wrong).

A forty-year-old woman is admitted to the emergency room at George Washington University Hospital after a serious car accident. Hospital staff attempt to obtain the woman's emergency contact information, in order to have someone come to the hospital as a source of support. The woman refuses to provide any emergency contact information, stating that she does not need, want, or have anyone whom she wishes to be contacted on her behalf. The woman further adds that she prefers to live a solitary life that is not dependent on others. Based on this limited information, which of the following best describes her interactions with her primary caregivers during childhood and her resulting attachment style? A) A secure attachment style resulting from interactions with loving and supportive parents who were available and comforting when needed. B) A disorganized attachment style resulting from interactions with parents that were sometimes positive and at other times harsh and rejecting. C) An anxious-resistant insecure attachment style resulting from interactions between parent and child that leave the child feeling uncertain about the availability of the parent during times of need. D) An anxious-avoidant insecure attachment style resulting from rejection of the child by the parent during times of need.

D. The woman's reluctance to form bonds and attachments with others and preference to live a solitary life may be indicative of her having formed an anxious-avoidant insecure attachment style during childhood. This style is the result of the individual's primary care giver constantly rejecting or rebuffing them when they sought comfort or protection. Eventually, because of these repeated rejections, they come to expect the same from other individuals they encounter, thus at the extreme, prefer to live a life free of any attachment to others, much like the woman described (choice D is correct). If the woman had established a secure attachment style during childhood, she would likely have numerous individuals with whom she has bonded and formed strong relationships. Thus, she would have individuals in her life from which she would like to receive comfort/support (choice A is wrong). This woman does appear to have difficulty trusting others, which could be indicative of disorganized attachment. However, disorganized attachment usually does not lead to disavowing of relationships and significant connections to others. Rather, the quest to form connections with others is hampered by poor social skills and inability to regulate emotions (choice B is wrong). An anxious-resistant insecure attachment style usually results from parents that are sometimes supportive and comforting and other times absent and unavailable, leaving the child uncertain about the parent's availability. This leads to preoccupation in adulthood with being abandoned by significant others, thus being hesitant to form relationships or when in relationships, displaying clingy behaviors. The woman in question, under this attachment style, would likely be very motivated to have significant people in her life be contacted and available during such a time of need (choice C is wrong).

Because researchers chose to use a questionnaire, which of the following was missing from this study? *From passage: Researchers distributed a questionnaire to Norwegian university students in 2012 that 1,000 (600 women and 400 men, each with an average age of 21) completed and returned. This questionnaire contained descriptive questions about both short-term and long-term relationship age and gender preferences for both ideal partners and actual partners. Figure 1 shows the results for the differences in age between participants and their ideal and actual long-term partners. A) Dependent variables B) Experimental method C) Independent variables D) A control group

D. Using a questionnaire meant that the researchers did not utilize a control group, which meant they did not have two groups participating simultaneously, one of whom would answer "normally," which is simply not possible with this methodology (choice D is correct). The dependent variables in this study were the age and gender preferences of students polled in the questionnaire—what the researchers were looking to study (choice A is wrong). The experimental method or scientific method is ideally present in all scientific research, where one variable is held constant and another is changed in order to study the effect. In this case, it is looking at the age and gender preferences for mating of a particular age and gender group of people (choice B is wrong). The independent variable here is the group of students polled—those between the ages of 18 and 30, identifying as male and female (choice C is wrong).

Great description of the sleep cycle:

During stage 3 sleep, EEG measures demonstrate delta waves, which are high amplitude, low frequency waves (choice C is correct). During stage 1 sleep, EEG measures demonstrate theta waves: waves of low to moderate intensity and intermediate frequency; alpha waves are detected on EEG during restful waking periods (choice A is wrong). During stage 2 sleep, EEG measures demonstrate two distinct wave patterns; although theta waves are still detected, these waves are intermixed with K-complexes and sleep spindles. Beta waves are detected on EEG during normal waking consciousness (choice B is wrong). During REM sleep, EEG measures demonstrate waves that most resemble the beta waves seen during wakefulness; however, the waves in REM sleep are more jagged in appearance than beta waves, which are also low intensity, but high frequency. Delta waves are characteristics of deep sleep (stages 3 and 4), not REM sleep, which while deep, is known as "paradoxical sleep," because the EEG measures most resemble that of wakefulness (choice D is wrong).

______ is the area of highest visual acuity in the eye.

Fovea contains a high concentration of cones

In Erikson's psychosocial stages of development, which stage occurs when it involves testing limits, having a sense of self, clarifying identity, goals, and life meaning?

Identity vs Role Confusion **Identity is one's sense of self and values

In Erikson's psychosocial stages of development, which stage occurs when it involves needing to understand world, gender-role identity, succeed in school, and competence?

Industry vs Inferiority

In Erikson's psychosocial stages of development, which stage occurs when it involves purpose, as well as needing to make decisions?

Initiative vs Guilt

In Erikson's psychosocial stages of development, which stage occurs when it involves needed to form relations?

Intimacy vs Isolation

What do Neobehaviorists believe?

Neobehaviorists believe that behavior can be modified by rewards or punishments **neobehaviorists believe that the most effective way to modify behavior is with positive reinforcement Therefore, if someone exhibits a desired behavior, providing a reward after that behavior will encourage the behavior to happen again (positive reinforcement). When a person exhibits an undesirable behavior, the application of a punishment will discourage that behavior from happening again (positive punishment).

According to Freud, the ego is ruled by the ___________ principle, which employs logical thinking to control consciousness and the id.

Reality

_________ connects the cerebrum and spinal cord to the cerebellum.

Pons

____________ explains how attitudes are better at predicting general patterns of behavior, but cannot always account for specific behaviors.

Principle of Aggregation

Self-Determination Theory

Self-determination theory maintains that, throughout the lifespan and especially in older age, people seek autonomy, competence, and relatedness. --> Autonomy refers to viewing one's circumstances as self-endorsed (self-approval) --> competence refers to the expression of one's talents --> relatedness refers to a sense of being cared about by other people

T or F: Students who advocate for a position are more likely to support that position when asked about their opinions if the incentive received was minimal

TRUE Students who advocate for a position are more likely to support that position when asked about their opinions if the incentive received was minimal, as they are more likely to attribute their opinion change to internal reasons than external. Thus, cognitive dissonance causes them to shift their opinion

Freudian Personality Theory

The components of Freud's structural theory, which describes personality, are the Id, Ego, and Superego. The Id contains the basic drives or impulses the Superego contains the conscience which is the internalized approval of the authority (the father). The Ego is the 'self' which manages the conflicts between the other two

What's involved in the limbic system?

The limbic system is a system of connected structures buried deep within the cerebrum, including the: - hippocampus - amygdala - the olfactory bulbs - limbic lobe **where most emotional experience and processing takes place

Representative vs Availability Heuristic:

The representativeness heuristic occurs when we estimate the likelihood of something by comparing it to an existing prototype that already exists in our minds. Our prototype is what we think is the most relevant or typical example of something. For example, most people have a mental prototype of a doctor as a middle-aged white male; therefore, the patient who mistakes the young, African American female for the nurse instead of the surgeon is doing so because of the representativeness heuristic (choice A is correct) The availability heuristic occurs when we rely on immediate examples that come to mind when trying to make a decision or judgment. When you overestimate the probability or likelihood of something happening because you can think of examples of it happening, as the man who cancelled his trip due to recent news coverage of airplane crashes has done, the availability heuristic has occurred

ultimate attribution error

The ultimate attribution error occurs when negative behavior by a member of an out-group is attributed by an in-group member to the out-group member's characteristics (e.g., genetics, intelligence, personality, childhood background), while positive behavior by a member of an out-group is rationalized or explained away by a member of an in-group (e.g., good luck, special circumstances, etc.).

The primary neurotransmitter of the parasympathetic nervous system is __________.

acetylcholine

What are the catecholamine neurotransmitters?

dopamine norepinephrine epinephrine *secreted from adrenal medulla **in fight-or-flight

The ability to shift from one task to another is an example of executive functioning, which is associated with the ______ lobe

frontal

Mindguarding is an aspect of ______, in which people self-censor in order to retain group harmony or avoid "rocking the boat"

groupthink

Paul Ekman finds that six basic emotions can be nearly universally categorized across all human cultures; What are they?

happiness, sadness, surprise, fear, disgust, and anger

Conservation (as learned in the concrete operational stage of Piaget) can be realized through three insights. What are they?

identity, reversibility, and reciprocity - In identity, the child recognizes that the physical object is the same object, regardless of how it is manipulated - In reversibility, the child recognizes that a given manipulation can be reversed to give the object its initial appearance - in reciprocity, the child realizes that a manipulation of one dimension or aspect yields a corresponding change in another aspect. As a result, the manipulation is understood to change the original object rather than create a new one *The realization that an object continues to exist despite its disappearance from the visual field is known as object permanence (not related)

In Erikson's psychosocial stages of development, which stage occurs when it involves looking back on life with no regrets?

integrity vs despair

Ennui

is a mental state characterized by lethargy and apathy (i.e. lack of interest enthusiasm), often associated with depression; it is not a social effect

Primary progressive aphasia

is a rare disease that results in impaired language ability

The ostrich effect

it refers to an individual avoiding an apparently risky financial situation by pretending it does not exist

The feelings associated with a negative attitude toward an entire group are known as _________. *prejudice or discrimination

prejudice

Stress as seen through Hans Selye

supported by his research, the physiological response to stress is universal

In Erikson's psychosocial stages of development, which stage occurs when it involves physical and emotional needs met?

trust vs mistrust

In classical learning theory, the process of extinction entails the repeated presentation of the conditioned stimulus without the _______. As a result, the conditioned stimulus loses its excitatory power, and the frequency or intensity of the conditioned response decreases

unconditioned stimulus

When are stereotypes the weakest?

when individuals or groups are familiar with each other **they are strongest when individuals or groups are unfamiliar with each other or when they belong to different groups So, (example) If the investigators of a study wanted to allow for the maximum chance that negative stereotypes would not interfere with the interactions between the adolescents and older people in the experimental groups, they might pair adolescents with older people: whom the adolescents knew very well.


Kaugnay na mga set ng pag-aaral

Marketing 341 UNL Exam 4 Chapter 16

View Set

CISCO NetAcad CCNA 1 Chapter 9 Exam

View Set

ACCT 402 CH 5 HW Review Questions

View Set

Intro to psychology: study guide 2

View Set

Geo Chap 19: Wind and Arid Regions

View Set